SlideShare a Scribd company logo
TO GET ALL CHAPTERS EMAIL ME AT>>>>> donc8246@gmail.com
TEST BANK
Paramedic Care- Principles & Practice,
6th Edition, Volume 5 by Bledsoe, Chapters 1 - 16
TO GET ALL CHAPTERS EMAIL ME AT>>>>>
donc8246@gmail.com
Test Bank Paramedic Care: Principles & Practice V.5, 6e (Bledsoe)
Volume 5: Special Considerations and Operations
Table of contents
Chapter 1. Gynecology
Chapter 2. Obstetrics
Chapter 3. Neonatology
Chapter 4. Pediatrics
Chapter 5. Geriatrics
Chapter 6. Abuse, Neglect, and Assault
Chapter 7. The Challenged Patient
Chapter 8. Acute Interventions for the Chronic Care Patient
Chapter 9. Ground Ambulance Operations
Chapter 10. Air Medical Operations
Chapter 11. Multiple-Casualty Incidents and Incident Management
Chapter 12. Rescue Awareness and Operations
Chapter 13. Hazardous Materials
Chapter 14. Crime Scene Awareness
Chapter 15. Rural EMS
Chapter 16. Responding to Terrorist Acts
TO GET ALL CHAPTERS EMAIL ME AT>>>>>
donc8246@gmail.com
Paramedic Care: Principles & Practice V.5, 6e (Bledsoe)
Volume 5: Special Considerations and Operations
Chapter 1: Gynecology
1) The paramedic accurately describes the difference between endometritis and endometriosis
when she states:
A) "Endometriosis is an infection of the uterine lining, while endometritis occurs when
endometrial tissue is found outside the uterus."
B) "Endometriosis occurs in women under age 40, while endometriosis is more common in
women who are older."
C) "Endometritis is an infection of the uterine lining, while endometriosis occurs when
endometrial tissue is found outside the uterus."
D) "Endometritis patients can be transported code 2, while those with endometriosis should
always go code 3."
ANSWER: C
Diff: 2 Page Ref: 12
Standard: Medicine (Gynecology) Objective:
1
2) Your patient tells you that she is being treated for cystitis. You recognize that she is being
treated for:
A) ovarian cysts.
B) pelvic inflammatory disease.
C) a urinary tract infection.
D) an ectopic pregnancy.
ANSWER: C
Diff: 1 Page Ref: 12
Standard: Medicine (Gynecology) Objective:
1
3) Which of the following is TRUE of mittelschmerz?
TO GET ALL CHAPTERS EMAIL ME AT>>>>>
donc8246@gmail.com
A) It is typically located unilaterally in one of the upper abdominal quadrants.
B) It is usually accompanied by heavy vaginal bleeding.
TO GET ALL CHAPTERS EMAIL ME AT>>>>>
donc8246@gmail.com
C) It is associated with ovulation.
D) It is a sign of ectopic pregnancy.
ANSWER: C
Diff: 1 Page Ref: 12
Standard: Medicine (Gynecology)
Objective: 1
TO GET ALL CHAPTERS EMAIL ME AT>>>>>
donc8246@gmail.com
4) Menorrhagia is:
A) irregular cycles of menstruation.
B) painful menstruation.
C) absence of menstruation.
D) excessive menstrual flow.
ANSWER: D
Diff: 1 Page Ref: 13
Standard: Medicine (Gynecology)
Objective: 1
5) Mittelschmerz refers to which of the following?
A) Purulent vaginal discharge
B) False labor pains
C) Midcycle abdominal pain
D) Painful urination
ANSWER: C
Diff: 1 Page Ref: 12
Standard: Medicine (Gynecology)
Objective: 1
6) Which of the following occurs during the proliferative phase of the menstrual cycle?
A) Ovulation
B) An increase in uterine vascularity
C) Endometrial thickening
D) A drop in estrogen levels
ANSWER: C
Diff: 1 Page Ref: 6
Standard: Medicine (Gynecology)
Objective: 2
7) The innermost lining of the uterus is called the:
A) myometrium.
B) perimetrium.
TO GET ALL CHAPTERS EMAIL ME AT>>>>>
donc8246@gmail.com
C) endometrium
TO GET ALL CHAPTERS EMAIL ME AT>>>>>
donc8246@gmail.com
D) vasometrium.
ANSWER: C
Diff: 1 Page Ref: 5
Standard: Medicine (Gynecology)
Objective: 2
TO GET ALL CHAPTERS EMAIL ME AT>>>>>
donc8246@gmail.com
8) A 22-year-old woman presents in moderate distress, complaining of diffuse lower abdominal
pain. She states that the pain has become progressively worse for the past two weeks and she is
now unable to walk without an increase in pain. Your physical exam reveals severe pain with
palpation of the lower abdomen and the following vital signs: heart rate 102, blood pressure
118/74 mmHg, and respirations 20. Which of the following statements made by the patient would
most indicate the presence of pelvic inflammatory disease?
A) "My boyfriend was just diagnosed with chlamydia."
B) "I just had an IUD inserted."
C) "My last menstrual cycle was normal."
D) "I have chronic urinary tract infections."
ANSWER: A
Diff: 2 Page Ref: 11
Standard: Medicine (Gynecology) Objective:
3
9) A 20-year-old sexually active woman presents with severe right-side abdominal pain that radiates
to her back. She states that the pain came on sharply during intercourse about 15 minutes earlier,
and she reports a small amount of vaginal bleeding. She states that her menstrual cycles have been
irregular for the past 3 months. The most likely clinical diagnosis would be:
A) ruptured ectopic pregnancy.
B) ruptured ovarian cyst.
C) spontaneous abortion.
D) pelvic inflammatory disease.
ANSWER: B
Diff: 2 Page Ref: 11-12 Standard:
Medicine (Gynecology) Objective:
3
10) A 33-year-old woman presents with a low-grade fever and abdominal pain. She reports that
she noticed blood in her urine this morning. Which of the following questions would be most
helpful when trying to identify the underlying cause of this patient's symptoms?
A) "Have you noticed any foul-smelling discharge?"
B) "Do you have any pain or burning with urination?"
TO GET ALL CHAPTERS EMAIL ME AT>>>>>
donc8246@gmail.com
C) "Do you take birth control?"
TO GET ALL CHAPTERS EMAIL ME AT>>>>>
donc8246@gmail.com
D) "Have you vomited today?"
ANSWER: B
Diff: 3 Page Ref: 12
Standard: Medicine (Gynecology)
Objective: 3
TO GET ALL CHAPTERS EMAIL ME AT>>>>>
donc8246@gmail.com
11) A 19-year-old woman presents with severe lower abdominal pain, an oral temperature of
102.4°F, and skin that is pale and sweaty. She reports that she had an elective abortion 72 hours
earlier and has had bloody vaginal discharge ever since. Appropriate treatment for this patient
would include which of the following?
A) IV fluids, oxygen, and transport
B) Position of comfort, pain medication, and delayed transport
C) Detailed secondary exam and 12-lead ECG
D) Knee-chest position and rapid transport
ANSWER: A
Diff: 2 Page Ref: 13
Standard: Medicine (Gynecology)
Objective: 3
12) A 22-year-old woman presents with severe abdominal pain and signs of shock. When asked
whether she is pregnant, she states, "There's no way I'm pregnant, I have an IUD." Which of the
following is the most likely cause of her signs and symptoms?
A) Endometriosis
B) Pelvis inflammatory disease
C) Miscarriage
D) Ectopic pregnancy
ANSWER: D
Diff: 2 Page Ref: 12-13 Standard:
Medicine (Gynecology) Objective:
3
13) All of the following are associated with spontaneous abortion EXCEPT:
A) nontraumatic vaginal bleeding.
B) the passage of clots and tissue.
C) hypotension and fever.
D) cramping abdominal pain.
ANSWER: C
Diff: 2 Page Ref: 13
Standard: Medicine (Gynecology)
TO GET ALL CHAPTERS EMAIL ME AT>>>>>
donc8246@gmail.com
Objective: 3
TO GET ALL CHAPTERS EMAIL ME AT>>>>>
donc8246@gmail.com
14) Your patient is a 42-year-old woman who is alert and upset, complaining of vaginal bleeding.
She states that the bleeding began ten hours ago and is heavier than her normal menstrual flow.
Her last menstrual period was three months ago, G3P3, and there is no other significant
gynecologic history. Which of the following is the most likely cause of her signs and symptoms?
A) Ectopic pregnancy
B) Pelvic inflammatory disease
C) Menopause-related dysfunctional uterine bleeding
D) Spontaneous abortion
ANSWER: D
Diff: 2 Page Ref: 13
Standard: Medicine (Gynecology)
Objective: 3
15) A 34-year-old woman presents with complaints of vaginal bleeding and pain during intercourse.
Her last menstrual period was two weeks ago and was reportedly heavier than normal. She reports
that she had saturated two menstrual pads in the past two hours. You should:
A) assess vital signs and transport.
B) apply oxygen and start a large-bore IV.
C) advise her to insert a tampon to control the bleeding.
D) ask her if she has a family history of uterine cancer.
ANSWER: A
Diff: 2 Page Ref: 13
Standard: Medicine (Gynecology)
Objective: 4
16) A 16-year-old female patient presents with complaints of abdominal pain and dizziness. She is
pale and clammy, and reluctant to answer your questions regarding her sexual activity. Vital signs
are: blood pressure 92/62 mmHg, heart rate 102, and respirations 20. You should:
A) make it clear that she must be honest about her sexual history.
B) start a large-bore IV and transport emergently.
C) ask the mother if the patient might be pregnant.
D) transport in a position of comfort.
ANSWER: B
TO GET ALL CHAPTERS EMAIL ME AT>>>>>
donc8246@gmail.com
Diff: 2 Page Ref: 13
Standard: Medicine (Gynecology)
Objective: 4
TO GET ALL CHAPTERS EMAIL ME AT>>>>>
donc8246@gmail.com
17) Your patient is a 35-year-old woman who is complaining of severe abdominal pain in both
lower quadrants. She states that she had a tubal ligation two years ago. Which of the following
questions is appropriate to ask during your care of this patient?
A) "Have you ever had a sexually transmitted disease?"
B) "Do you have more than one sexual partner?"
C) "Have you ever had pelvic inflammatory disease (PID)?"
D) "When was your last menstrual period?"
ANSWER: D
Diff: 1 Page Ref: 13
Standard: Medicine (Gynecology) Objective:
4
18) Risk factors for ectopic pregnancy include all of the following EXCEPT:
A) use of an intrauterine device for birth control.
B) pelvic inflammatory disease.
C) previous ectopic pregnancies.
D) IV drug usage.
ANSWER: D
Diff: 2 Page Ref: 12-13 Standard:
Medicine (Gynecology) Objective:
3
19) You are called to the scene of a sexual assault. The patient is a 17-year-old female who is crying
inconsolably and withdraws when you attempt to touch her. Which of the following is the most
appropriate response?
A) Explain to the patient that she must allow you to examine her for injuries.
B) Ask the patient to describe how she was assaulted so you know where she is injured.
C) Tell the patient that you cannot help her if she won't allow you to touch her.
D) Explain to the patient that you will not touch her if she does not want you too.
ANSWER: D
Diff: 1 Page Ref: 14
Standard: Medicine (Gynecology) Objective:
5
TO GET ALL CHAPTERS EMAIL ME AT>>>>>
donc8246@gmail.com
20) A 35-year-old woman has been raped. She states repeatedly that she wants to change her
clothes before going to the hospital, and becomes hysterical when you advise her that she should
remain dressed to preserve evidence. You should:
A) allow her to change and carefully bag each item of clothing.
B) refuse to let her change her clothing, as it will destroy evidence.
C) allow her to change only her shirt, not her pants or undergarments.
D) promise to let her change as soon as you get to the hospital.
ANSWER: A
Diff: 2 Page Ref: 14
Standard: Medicine (Gynecology) Objective:
5
TO GET ALL CHAPTERS EMAIL ME AT>>>>>
donc8246@gmail.com
21) Which of the following is NOT an appropriate adaptation to make in the assessment and care
of a sexual assault victim?
A) Allow a friend to accompany the victim in the back of the ambulance.
B) Ask simple, closed-ended questions about the nature of the assault.
C) Keep the back of the ambulance well lit and warm.
D) Do not touch the patient unless it is necessary to obtain vital signs or examine injuries.
ANSWER: B
Diff: 2 Page Ref: 14
Standard: Medicine (Gynecology)
Objective: 5
22) A 35-year-old woman presents with abdominal pain that she describes as "cramping and dull."
She reports having multiple sexual partners in the past six months, and states that she has an IUD.
Vital signs are: blood pressure 118/76 mmHg, heart rate 88, and respirations 18. Appropriate care
of this patient includes:
A) cardiac monitoring and transport.
B) examining the genitalia for hemorrhage.
C) elevating the legs during transport.
D) position of comfort and routine ALS care.
ANSWER: D
Diff: 2 Page Ref: 13
Standard: Medicine (Gynecology)
Objective: 6
23) A 22-year-old woman presents with left-sided lower abdominal pain. She reports that her last
menstrual cycle ended approximately 10 days ago. Her blood pressure is normal and she rates the
pain as 6 on a scale of 10. You should:
A) palpate the abdomen.
B) administer oxygen.
C) withhold pain medication.
D) look for vaginal bleeding.
ANSWER: A
Diff: 2 Page Ref: 13
TO GET ALL CHAPTERS EMAIL ME AT>>>>>
donc8246@gmail.com
Standard: Medicine (Gynecology)
TO GET ALL CHAPTERS EMAIL ME AT>>>>>
donc8246@gmail.com
Objective: 6
TO GET ALL CHAPTERS EMAIL ME AT>>>>>
donc8246@gmail.com
24) Your patient is an 18-year-old woman who is alert and in moderate distress, complaining of
abdominal pain and light-headedness. She describes a four-week history of worsening unilateral
lower quadrant abdominal pain with onset of malaise, nausea, and vomiting this week. Today, she is
experiencing faintness and near-syncope with exertion. She denies pain or difficulty with urination.
Her last menstrual period was two weeks ago. She is sexually active and uses oral contraceptives.
Physical examination reveals marked tenderness and guarding with palpation of her abdomen. Her
skin is cool and diaphoretic. Her vital signs are: heart rate 121, blood pressure 90/58 mmHg,
respirations 18. The patient's presentation is most consistent with:
A) ectopic pregnancy.
B) pelvic inflammatory disease.
C) spontaneous abortion.
D) pyelonephritis.
ANSWER: A
Diff: 3 Page Ref: 12-13 Standard:
Medicine (Gynecology) Objective:
6
25) A 30-year-old woman presents with a fever of 103.5°F, pale skin, and an altered mental status.
She cries out and withdraws when you palpate her abdomen. Her husband reports that she had a
cervical biopsy three days earlier, and has be "feeling sick" ever since. Vital signs are: blood pressure
88/60 mmHg, heart rate 110, and respirations 22. You suspect:
A) sepsis secondary to endometritis.
B) hypovolemia secondary to miscarriage.
C) infection of fibroid tumors.
D) shock due to pelvic inflammatory disease.
ANSWER: A
Diff: 3 Page Ref: 12
Standard: Medicine (Gynecology) Objective:
6
26) A 26-year-old woman is complaining of severe abdominal pain and heavy vaginal bleeding. She
has used five sanitary napkins in the past hour. Her last menstrual period was six weeks ago, and
she describes a history of irregular periods, but never to this extent. Management of this patient
TO GET ALL CHAPTERS EMAIL ME AT>>>>>
donc8246@gmail.com
should include:
TO GET ALL CHAPTERS EMAIL ME AT>>>>>
donc8246@gmail.com
A) IV dextrose.
B) oral antiemetic.
C) IM epinephrine.
D) IV fluids.
ANSWER:
D
Diff: 2 Page Ref: 13
Standard: Medicine (Gynecology)
Objective: 6
TO GET ALL CHAPTERS EMAIL ME AT>>>>>
donc8246@gmail.com
27) Your patient is a 44-year-old woman who is alert and in mild distress. She states that she had an
acute onset of sharp, right lower quadrant abdominal pain last evening while having intercourse and
that the pain has not subsided. Physical examination reveals tenderness with palpation to the lower
right abdominal quadrant. Vital signs are: heart rate 98, respirations 16, blood pressure 116/78
mmHg. She reports she had a tubal ligation ten years ago and that she has several small fibroid
tumors. Which of the following is the most likely diagnosis for this patient?
A) Ectopic pregnancy
B) Pelvic inflammatory disease
C) Mittelschmerz
D) Ruptured ovarian cyst
ANSWER: D
Diff: 2 Page Ref: 11-12 Standard:
Medicine (Gynecology) Objective:
3
28) Your patient is a transgender man complaining of severe lower abdominal pain. He states that
he is in the process of transitioning from female to male, and that he takes hormone supplements.
When evaluating this patient, which of the following questions is most appropriate?
A) "Are you planning to have gender reassignment surgery?"
B) "When was your last menstrual cycle?"
C) "How often do you use illegal drugs?"
D) "Are you usually sexually active with men, or with women?"
ANSWER: B
Diff: 3 Page Ref: 13
Standard: Medicine (Gynecology)
Objective: 6
29) A 27-year-old woman reports significant vaginal bleeding. She states that she is 11 weeks
pregnant and awoke this morning to heavy bleeding including the passage of clots. She is crying
and very upset, stating, "I can't lose this baby—I can't live with that." When caring for this patient,
you should:
A) ask her if she has ever had a miscarriage.
B) perform a vaginal exam to determine if she is still bleeding.
TO GET ALL CHAPTERS EMAIL ME AT>>>>>
donc8246@gmail.com
C) transport the patient in a position of comfort and provide emotional support.
TO GET ALL CHAPTERS EMAIL ME AT>>>>>
donc8246@gmail.com
D) inform the patient's husband that she may be having a miscarriage.
ANSWER: C
Diff: 2 Page Ref: 13
Standard: Medicine (Gynecology)
Objective: 6
TO GET ALL CHAPTERS EMAIL ME AT>>>>>
donc8246@gmail.com
30) A 30-year-old woman has right-sided abdominal pain and is hypotensive. You suspect she
may have a ruptured ectopic pregnancy. You recognize:
A) life-threatening hemorrhagic shock and the need for fluid resuscitation.
B) potentially dangerous obstructive shock that requires rapid transport.
C) a common medical condition that affects many women each year.
D) a very rare disease process that requires immediate surgery to save the pregnancy.
ANSWER: A
Diff: 3 Page Ref: 13
Standard: Medicine (Gynecology) Objective:
6
31) Which of the following is NOT an appropriate way to control vaginal bleeding in the
prehospital setting?
A) Placing sterile gauze over the opening of the vagina
B) Using a tampon
C) Using a sanitary napkin
D) Placing an absorbent pad under the patient
ANSWER: B
Diff: 1 Page Ref: 13
Standard: Medicine (Gynecology) Objective:
6
32) Your patient is a 36-year-old woman who is alert and complaining of abdominal pain. She
states that she is having her period and that this pain is "much different than the cramps I usually
get." She describes the pain as achy throughout her pelvis and lower abdomen. She says that this
has occurred the past three menstrual cycles and that she has experienced dyspareunia and spotting
over the same period. She is G2P2 and has no other significant gynecologic history. Physical
examination reveals pain with palpation over her entire abdomen; her skin is warm and dry. Vital
signs are: heart rate 84, blood pressure 124/76 mmHg, respiration 12, SpO2 = 99%. Which of the
following is the most likely diagnosis for this patient?
A) Uterine fibroids
B) Endometriosis
C) Primary dysmenorrhea
TO GET ALL CHAPTERS EMAIL ME AT>>>>>
donc8246@gmail.com
D) Polycystic ovary disease
TO GET ALL CHAPTERS EMAIL ME AT>>>>>
donc8246@gmail.com
ANSWER: B
Diff: 2 Page Ref: 12
Standard: Medicine (Gynecology)
Objective: 3
TO GET ALL CHAPTERS EMAIL ME AT>>>>>
donc8246@gmail.com
Chapter 2 Obstetrics
1) Just before an infant is delivered, the fetal scalp can be seen at the vaginal opening during
each contraction. This is referred to as:
A) effacement.
B) crowning.
C) dilation.
D) presentation.
ANSWER: B
Diff: 1 Page Ref: 28
Standard: Special Patient Populations (Obstetrics)
Objective: 1
2) The blood-rich structure that serves as a lifeline for the developing fetus is called the:
A) uterus.
B) cervix.
C) placenta.
D) amniotic sac.
ANSWER: C
Diff: 1 Page Ref: 20
Standard: Special Patient Populations (Obstetrics)
Objective: 1
3) The medically appropriate term for the projected birth date of a baby is:
A) probable date of delivery.
B) expected date of labor.
C) predicted delivery date.
D) estimated date of confinement.
ANSWER: D
Diff: 1 Page Ref: 23
Standard: Special Patient Populations (Obstetrics)
Objective: 1
TO GET ALL CHAPTERS EMAIL ME AT>>>>>
donc8246@gmail.com
4) The term "puerperium" refers to:
TO GET ALL CHAPTERS EMAIL ME AT>>>>>
donc8246@gmail.com
A) the delivery of the afterbirth.
B) a specific fertility treatment.
C) the time surrounding delivery.
D) a technique used to delay contractions.
ANSWER: C
Diff: 1 Page Ref: 36
Standard: Special Patient Populations (Obstetrics)
Objective: 1
TO GET ALL CHAPTERS EMAIL ME AT>>>>>
donc8246@gmail.com
5) You are caring for a woman who is 32 weeks pregnant. While assessing her breathing, you
notice that her rib cage expands visibly with each breath and that her tidal volume appears to be
deep. You recognize:
A) signs of a severe respiratory condition requiring immediate intervention.
B) an increase in tidal volume that is normal during pregnancy.
C) signs of obstructive shock.
D) an increase in oxygen demand due to an obstetrical emergency.
ANSWER: B
Diff: 2 Page Ref: 21
Standard: Special Patient Populations (Obstetrics) Objective:
2
6) A 27-year-old pregnant woman was the restrained driver of a vehicle that hit the guardrail at a
high rate of speed. She sustained a large laceration to the upper thigh and has lost a significant
amount of blood. Vital signs are: heart rate 102, blood pressure 118/78 mmHg, respirations 18, and
her skin is pale and clammy. Which of the following BEST explains this patient's presentation?
A) Maternal blood volume increases during pregnancy, allowing vital signs to remain normal
despite significant blood loss.
B) Fetal oxygen demand increases after trauma, causing maternal blood pressure and heart rate to
increase immediately after injury.
C) Maternal blood volume decreases during pregnancy, amplifying the effects of epinephrine
and allowing for the maintenance of blood pressure.
D) Maternal vital signs remain normal following trauma to ensure the survival of the developing
fetus.
ANSWER: A
Diff: 2 Page Ref: 22
Standard: Special Patient Populations (Obstetrics) Objective:
2
7) A 30-year-old woman complains of swelling in both of her legs. She is 32 weeks pregnant,
and states that she has also developed varicose veins over the past few weeks. Which of the
following BEST explains the cause of this patient's signs and symptoms?
A) The gravid uterus compresses the superior vena cava, decreasing venous return to the heart.
TO GET ALL CHAPTERS EMAIL ME AT>>>>>
donc8246@gmail.com
B) Changes in the endocrine system cause peripheral edema during the late stages of pregnancy.
C) The gravid uterus compresses the pelvic and femoral vessels, decreasing venous return and
causing venous stasis.
D) Changes in kidney function alter the normal electrolyte balance, causing tissue edema in the
legs and feet.
ANSWER: C
Diff: 2 Page Ref: 22
Standard: Special Patient Populations (Obstetrics) Objective:
2
TO GET ALL CHAPTERS EMAIL ME AT>>>>>
donc8246@gmail.com
8) During which period of development is the fetus most susceptible to damage from maternal
exposure to toxins, such as alcohol and tobacco?
A) 1-8 weeks
B) 16-20 weeks
C) 8-12 weeks
D) 20-40 weeks
ANSWER: C
Diff: 2 Page Ref: 24
Standard: Special Patient Populations (Obstetrics) Objective:
2
9) Which structure allows oxygenated maternal blood to bypass the uninflated lungs of the
developing fetus?
A) Ductus venosus
B) Foramen ovale
C) Umbilical artery
D) Ductus arteriosus
ANSWER: A
Diff: 2 Page Ref: 25
Standard: Special Patient Populations (Obstetrics) Objective:
2
10) A 38-year-old pregnant woman called 911 after she developed a "splitting headache" unrelieved
by over-the-counter analgesics. She states that she has a previous diagnosis of diabetes that is
typically well controlled with insulin injections. This patient's history of diabetes makes her more
likely to develop which of the following pregnancy-related conditions?
A) Preeclampsia
B) Cerebral aneurysm
C) Congestive heart failure
D) Stroke
ANSWER:
A
TO GET ALL CHAPTERS EMAIL ME AT>>>>>
donc8246@gmail.com
Diff: 2 Page Ref: 26
Standard: Special Patient Populations (Obstetrics)
TO GET ALL CHAPTERS EMAIL ME AT>>>>>
donc8246@gmail.com
Objective: 3
TO GET ALL CHAPTERS EMAIL ME AT>>>>>
donc8246@gmail.com
11) A 29-year-old woman is 28 weeks pregnant. She complains of nausea, abdominal pain, and
right shoulder pain. She states she vomited once, which caused temporary relief; however, the pain
returned shortly afterward. She has no pertinent medical history or allergies, has stable vital signs,
and states that she ate a cheeseburger about 20 minutes prior to the start of her pain. The most
likely cause of this patient's presentation is:
A) food poisoning.
B) cholecystitis.
C) hyperemesis gravidarum.
D) appendicitis.
ANSWER: B
Diff: 2 Page Ref: 29
Standard: Special Patient Populations (Obstetrics)
Objective: 3
12) You are caring for a 42-year-old woman who is 32 weeks pregnant with twins. She is
complaining of a severe headache and blurred vision, and lives in a rural area approximately 30
miles from the nearest ED. Your physical exam reveals significant peripheral edema and the
following vital signs: heart rate 98, blood pressure 156/98 mmHg, respirations 18. The patient
states she has a history of hypertension, but has not taken her prescribed medications because
"they aren't safe for the babies." Appropriate treatment for this patient includes:
A) administration of nitroglycerine and delayed transport.
B) intravenous calcium chloride and 12-lead ECG.
C) administration of aspirin and a prehospital stroke assessment.
D) intravenous magnesium sulfate and rapid transport.
ANSWER: D
Diff: 3 Page Ref: 32-33
Standard: Special Patient Populations (Obstetrics)
Objective: 3
13) A 24-year-old pregnant woman is actively seizing. Her partner reports that she complained of
abdominal pain approximately 10 minutes ago, and then began seizing. He reports that the
seizure activity has been consistent for the past 10 minutes, and that she has no significant
medical history. You should:
TO GET ALL CHAPTERS EMAIL ME AT>>>>>
donc8246@gmail.com
A) apply a nasal cannula, assess blood glucose level, and transport in the left lateral position.
TO GET ALL CHAPTERS EMAIL ME AT>>>>>
donc8246@gmail.com
B) insert a nasal airway, administer midazolam, and obtain vital signs.
C) apply high-flow oxygen, administer magnesium sulfate, and transport emergently.
D) suction the airway, obtain IV access, and assess blood pressure.
ANSWER: C
Diff: 2 Page Ref: 32-33
Standard: Special Patient Populations (Obstetrics)
Objective: 3
TO GET ALL CHAPTERS EMAIL ME AT>>>>>
donc8246@gmail.com
14) A 25-year-old woman presents complaining of heavy vaginal bleeding accompanied by
cramping abdominal pain and backache. She can't stop crying and reports that she is 11 weeks
pregnant with her first child. Your physical exam reveals what appears to be the fetus and
umbilical cord passing through the vaginal opening; however, the placenta has not been passed.
Appropriate care for this patient includes:
A) clamping and cutting the umbilical cord, wrapping fetal material in linen, and providing
emotional support for the patient.
B) leaving the fetus untouched and transporting the patient in a position of comfort.
C) oxygen, 1000 mL fluid bolus, and rapid transport.
D) disposing of the fetal material in a biohazard bag, left lateral position, and providing
emotional support for the patient.
ANSWER: A
Diff: 3 Page Ref: 29
Standard: Special Patient Populations (Obstetrics) Objective:
4
15) You are caring for a patient who is 36 weeks pregnant. She states that she has been experiencing
vaginal bleeding after intercourse, and denies any associated pain. Vital signs are: heart rate 84,
respirations 18, blood pressure 102/72 mmHg, and warm, dry skin. The most likely cause of this
patient's vaginal bleeding is:
A) spontaneous abortion.
B) placenta previa.
C) abruptio placentae.
D) false labor.
ANSWER: B
Diff: 2 Page Ref: 31
Standard: Special Patient Populations (Obstetrics) Objective:
3
16) A 37-year-old, obviously pregnant woman complains of sharp, tearing abdominal pain. She
reports that she is pregnant with her fifth child, but states she cannot remember the exact due
date. The patient has a history of drug use and states that she last smoked crack cocaine
approximately 30 minutes ago. You suspect:
TO GET ALL CHAPTERS EMAIL ME AT>>>>>
donc8246@gmail.com
A) spontaneous abortion.
TO GET ALL CHAPTERS EMAIL ME AT>>>>>
donc8246@gmail.com
B) placenta previa.
C) abruptio placentae.
D) false labor.
ANSWER: C
Diff: 2 Page Ref: 31-32
Standard: Special Patient Populations (Obstetrics)
Objective: 3
TO GET ALL CHAPTERS EMAIL ME AT>>>>>
donc8246@gmail.com
17) A pregnant woman is complaining of a severe headache and "feeling ill." Her vital signs are:
heart rate 92, blood pressure 120/82 mmHg, and respiratory rate of 16. Which of the following
questions would be most helpful in identifying a possible pregnancy-related emergency?
A) "Do you have a history of migraine headaches?"
B) "Have you been experiencing morning sickness or excessive vomiting today?"
C) "When was the last time you saw your doctor?"
D) "What has been a typical blood pressure for you during this pregnancy?"
ANSWER: D
Diff: 2 Page Ref: 33
Standard: Special Patient Populations (Obstetrics) Objective:
4
18) Which of the following techniques is an appropriate method of estimating the gestational
age?
A) Measuring the circumference of the woman's abdomen; each inch corresponds to one week of
pregnancy.
B) Palpate the abdomen and estimate gestational age based on the size of the developing fetus.
C) Measure the height of the fundus; each centimeter corresponds to one week of pregnancy.
D) Palpate the abdomen; if fetal movement is felt then the pregnancy is at least 14 weeks.
ANSWER: C
Diff: 2 Page Ref: 47
Standard: Special Patient Populations (Obstetrics) Objective:
4
19) When assessing the vital signs of a pregnant patient, it is generally most appropriate to have
the patient in which position?
A) Supine
B) Left lateral recumbent
C) Semi-Fowler's
D) Standing
ANSWER:
B
TO GET ALL CHAPTERS EMAIL ME AT>>>>>
donc8246@gmail.com
Diff: 1 Page Ref: 27
Standard: Special Patient Populations (Obstetrics)
TO GET ALL CHAPTERS EMAIL ME AT>>>>>
donc8246@gmail.com
Objective: 4
20) You are caring for a 19-year-old woman who states that she is 32 weeks pregnant. She is
experiencing cramping abdominal pain every six to seven minutes and reports a brief period of
fluid discharge from her vagina approximately 15 minutes ago. You should:
A) provide routine BLS care and transport.
B) begin a fluid bolus and transport code 3.
C) administer ondansetron and reassess vital signs.
D) apply oxygen and perform a physical exam.
ANSWER: B
Diff: 2 Page Ref: 35-36
Standard: Special Patient Populations (Obstetrics) Objective:
5
TO GET ALL CHAPTERS EMAIL ME AT>>>>>
donc8246@gmail.com
21) The second stage of labor begins:
A) with the complete dilation of the cervix.
B) at the onset of contractions.
C) when the baby is delivered.
D) at the rupture of membranes.
ANSWER: A
Diff: 2 Page Ref: 36
Standard: Special Patient Populations (Obstetrics)
Objective: 6
22) You are caring for a woman in labor. She has had three previous deliveries and states that her
contractions are approximately 3 minutes apart. Physical exam reveals a bulging of tissue at the
vaginal opening with each contraction; however, the baby's head is not yet visible. You should:
A) encourage the mother to breathe and ready her for transport.
B) prepare for imminent delivery.
C) place the mother in the knee-chest position.
D) perform an internal exam to see how much the cervix is dilated.
ANSWER: B
Diff: 2 Page Ref: 38
Standard: Special Patient Populations (Obstetrics)
Objective: 6
23) What is the appropriate sequence of care for a newborn following a normal delivery?
A) Suction the nose, dry the infant, and cut the cord.
B) Dry the infant, cut the cord, and place infant on mother's stomach.
C) Suction the mouth, deliver the placenta, and cut the cord.
D) Apply oxygen, dry the infant, and give the infant to the mother.
ANSWER: B
Diff: 2 Page Ref: 39, 40
Standard: Special Patient Populations (Obstetrics)
Objective: 7
TO GET ALL CHAPTERS EMAIL ME AT>>>>>
donc8246@gmail.com
24) A newborn has just been delivered. He is centrally pink with pale extremities, has a heart rate of
110, and is actively crying and moving. The appropriate APGAR score for this infant is:
TO GET ALL CHAPTERS EMAIL ME AT>>>>>
donc8246@gmail.com
A) 6.
B) 7.
C) 8.
D) 9.
ANSWER:
D
Diff: 2 Page Ref: 41
Standard: Special Patient Populations (Obstetrics)
Objective: 7
TO GET ALL CHAPTERS EMAIL ME AT>>>>>
donc8246@gmail.com
25) A newborn is listless and has a heart rate of 50 beats per minute after 30 seconds of
stimulation. You should FIRST:
A) initiate positive pressure ventilation.
B) begin chest compressions.
C) obtain IV access.
D) apply the AED.
ANSWER: A
Diff: 2 Page Ref: 41
Standard: Special Patient Populations (Obstetrics)
Objective: 7
26) A 42-year-old woman is 38 weeks pregnant and in active labor. While assessing for crowning,
you notice that although the top of the baby's head is visible, a loop of the umbilical cord is
protruding from the vagina. You should:
A) gently attempt to reinsert the cord into the vaginal canal.
B) encourage the mother to push while gently pulling traction.
C) clamp and cut the presenting section of the umbilical cord.
D) insert two fingers to raise the head of the baby off the cord.
ANSWER: D
Diff: 2 Page Ref: 43
Standard: Special Patient Populations (Obstetrics)
Objective: 8
27) You are assisting a mother with an out-of-hospital delivery. After the head delivers, it
immediately retracts back into the perineum. You should:
A) transport in the knee-chest position.
B) instruct the mother to drop her buttocks off the end of the bed.
C) gently pull downward on the infant's head.
D) instruct the mother to avoid pushing if possible.
ANSWER: B
Diff: 2 Page Ref: 43
Standard: Special Patient Populations (Obstetrics)
TO GET ALL CHAPTERS EMAIL ME AT>>>>>
donc8246@gmail.com
Objective: 8
TO GET ALL CHAPTERS EMAIL ME AT>>>>>
donc8246@gmail.com
28) During delivery, you notice a yellowish-green fluid on the baby's head and face. You
recognize:
A) a normal side effect of delivery.
B) that emergency transport is required.
C) a sign of fetal hypoxia.
D) that the infant will require resuscitation.
ANSWER: C
Diff: 2 Page Ref: 45
Standard: Special Patient Populations (Obstetrics)
Objective: 8
TO GET ALL CHAPTERS EMAIL ME AT>>>>>
donc8246@gmail.com
29) You have just delivered a healthy newborn to a 35-year-old woman. There is a steady flow of
blood from the vagina and her uterus feels soft upon palpation. Her vital signs are: heart rate 122,
respirations 20, blood pressure 90/60 mmHg. Appropriate management of this patient includes:
A) sanitary pads placed over the vagina and continued monitoring.
B) two large-bore IVs and oxytocin administration.
C) Trendelenberg position and administration of magnesium sulfate.
D) low-flow oxygen and position of comfort.
ANSWER: B
Diff: 2 Page Ref: 45
Standard: Special Patient Populations (Obstetrics) Objective:
8
30) A 32-year-old woman was the restrained passenger of a vehicle that hit a tree at a high rate of
speed. She is 34 weeks pregnant and complains of excruciating abdominal pain. She is cool and
diaphoretic and has a faint and rapid radial pulse. You suspect:
A) traumatic onset of labor.
B) uterine inversion.
C) uterine rupture.
D) premature rupture of membranes.
ANSWER: C
Diff: 2 Page Ref: 46
Standard: Special Patient Populations (Obstetrics) Objective:
8
31) A woman late in her pregnancy is in cardiac arrest. When resuscitating this patient, it is
appropriate to:
A) compress at a rate of at least 120 per minute.
B) use a mechanical compression device.
C) lift and push the gravid uterus to the left.
D) assess for imminent delivery.
ANSWER: C
Diff: 2 Page Ref: 47-48
TO GET ALL CHAPTERS EMAIL ME AT>>>>>
donc8246@gmail.com
Standard: Special Patient Populations (Obstetrics) Objective:
8
TO GET ALL CHAPTERS EMAIL ME AT>>>>>
donc8246@gmail.com
32) You have just delivered an infant at 33 weeks gestation. The infant is blue and limp and has a
brachial pulse of 70 beats per minute. What is the appropriate sequence of care for this infant?
A) Keep warm and dry, chest compressions, clamp and cut the cord.
B) Keep warm and dry, clamp and cut the cord, positive pressure ventilations.
C) Clamp and cut the cord, positive pressure ventilations, keep warm and dry.
D) Clamp and cut the cord, chest compressions, positive pressure ventilations.
ANSWER: B
Diff: 2 Page Ref: 41
Standard: Special Patient Populations (Obstetrics)
Objective: 8
TO GET ALL CHAPTERS EMAIL ME AT>>>>>
donc8246@gmail.com
33) A 21-year-old woman is complaining of abdominal pain. She says she is 25 weeks pregnant and
has been actively trying to stop using methamphetamines, but slipped and used earlier today. Her
abdominal pain comes every 8 to 10 minutes and is described as sharp and cramping. You suspect:
A) hypertensive disorder of pregnancy.
B) abruptio placentae.
C) toxic shock syndrome.
D) preterm labor.
ANSWER: D
Diff: 2 Page Ref: 35
Standard: Special Patient Populations (Obstetrics)
Objective: 8
34) When is the ideal time to assess a neonate for the APGAR score?
A) 2 to 3 minutes after delivery
B) 1 minute and 5 minutes after delivery
C) 5 to 10 minutes after delivery
D) 2 minutes and 10 minutes after delivery
ANSWER: B
Diff: 2 Page Ref: 40
Standard: Special Patient Populations (Obstetrics)
Objective: 7
Chapter 3 Neonatology
1) In some cases, the spinal cord and associated structures of a newborn may be exposed. This
abnormality is called a:
A) Tetralogy of Fallot.
B) meningomyelocele.
C) omphalocele.
D) choanal atresia.
ANSWER: B
Diff: 2 Page Ref: 59
TO GET ALL CHAPTERS EMAIL ME AT>>>>>
donc8246@gmail.com
Standard: Special Patient Populations (Neonatal Care)
TO GET ALL CHAPTERS EMAIL ME AT>>>>>
donc8246@gmail.com
Objective: 1
2) Which of the following best describes Pierre Robin syndrome?
A) A congenital condition characterized by a small jaw and large tongue in conjunction with a
cleft palate
B) A condition in which the ductus arteriosis fails to close completely
C) Repeat febrile seizures within the first 6 months of life
D) Persistent pneumonia following meconium aspiration
ANSWER: A
Diff: 2 Page Ref: 59
Standard: Special Patient Populations (Neonatal Care)
Objective: 1
TO GET ALL CHAPTERS EMAIL ME AT>>>>>
donc8246@gmail.com
3) Which of the following is TRUE regarding vaginal delivery?
A) Infants delivered vaginally are at greater risk of post-delivery complications.
B) The compression of the infant's chest during vaginal delivery aids in the removal of fluid
from the fetal lungs.
C) Vaginal delivery stimulates the production of insulin in the newborn.
D) The foramen ovale close more successfully in newborns delivered vaginally.
ANSWER: B
Diff: 2 Page Ref: 55
Standard: Special Patient Populations (Neonatal Care)
Objective: 2
4) Factors that stimulate the newborn to take the first breath include all of the following
EXCEPT:
A) hypothermia.
B) hypoxia.
C) hyperglycemia.
D) acidosis.
ANSWER:
C
Diff: 2 Page Ref: 55
Standard: Special Patient Populations (Neonatal Care)
Objective: 3
5) You are caring for a newborn who had prolonged respiratory distress and hypoxia following
birth. You recognize that continued hypoxia and subsequent severe acidosis can cause:
A) refractory hypotension.
B) rapid development of fetal alveoli to compensate for hypoxia.
C) persistent fetal circulation.
D) the formation of congenital heart defects.
ANSWER: C
Diff: 3 Page Ref: 55
Standard: Special Patient Populations (Neonatal Care)
TO GET ALL CHAPTERS EMAIL ME AT>>>>>
donc8246@gmail.com
Objective: 2
TO GET ALL CHAPTERS EMAIL ME AT>>>>>
donc8246@gmail.com
6) You have been called to the residence of a five-day-old infant. His parents are
concernedbecause he sleeps about 18 hours a day. They took a rectal temperature and got a
reading of100°F. Which of the following is the best course of action?
A) Administer a 10 mL/kg fluid bolus.
B) Undress the infant to his diaper and sponge him with tepid water.
C) Reassure the parents that the infant's sleep patterns and temperature are normal.
D) Contact medical control to order acetaminophen.
ANSWER: C
Diff: 2 Page Ref: 59-60
Standard: Special Patient Populations (Neonatal Care) Objective:
3
TO GET ALL CHAPTERS EMAIL ME AT>>>>>
donc8246@gmail.com
7) You have just assisted in the delivery of a full-term infant in the back of the ambulance. Ten
minutes after delivery, you obtain vital signs on the newborn and find the following: heart rate
120, respirations 54, and SpO2 of 90%. You recognize:
A) the need for immediate supplemental oxygen.
B) appropriate vital signs for this infant.
C) tachycardia and the possibility of shock.
D) tachypnea and possible respiratory distress.
ANSWER: C
Diff: 2 Page Ref: 60
Standard: Special Patient Populations (Neonatal Care) Objective:
5
8) Which of the following is NOT an option for prehospital vascular access in the newborn?
A) Cannulation of a peripheral vein in an extremity
B) Intraosseous needle placed in the proximal tibia
C) Femoral vein cannulation
D) Umbilical vein cannulation
ANSWER: C
Diff: 2 Page Ref: 67
Standard: Special Patient Populations (Neonatal Care) Objective:
7
9) Which of the following is NOT a recommended method for assessing the heart rate of a
newborn?
A) Palpating the carotid pulse
B) Palpate the umbilical pulse
C) Auscultate heart sounds
D) Palpate a femoral pulse
ANSWER: A
Diff: 2 Page Ref: 64-65
Standard: Special Patient Populations (Neonatal Care) Objective:
5
TO GET ALL CHAPTERS EMAIL ME AT>>>>>
donc8246@gmail.com
10) Which of the following statements is TRUE regarding the cardiac output of a neonate?
TO GET ALL CHAPTERS EMAIL ME AT>>>>>
donc8246@gmail.com
A) Cardiac output doubles within the first few hours of life.
B) The cardiac output of a neonate is strongly influenced by body temperature.
C) Neonates do not readily alter their pulse rate; cardiac output depends mainly on stroke
volume.
D) Newborns have a fixed stroke volume; cardiac output depends mostly on heart rate.
ANSWER: D
Diff: 2 Page Ref: 64
Standard: Special Patient Populations (Neonatal Care) Objective:
3
TO GET ALL CHAPTERS EMAIL ME AT>>>>>
donc8246@gmail.com
11) A newborn has just been delivered. You notice a greenish-brown substance on the infant, and
observe a vigorous cry and active movement. What is the appropriate way to manage this patient?
A) Prepare to suction the trachea with an endotracheal tube and meconium aspirator.
B) Using a bulb syringe, suction the nose and then the mouth.
C) Rub the infant with a dry towel and place on the mother's chest.
D) Lay the infant supine and place padding behind the shoulders.
ANSWER: C
Diff: 2 Page Ref: 70-71
Standard: Special Patient Populations (Neonatal Care)
Objective: 3
12) At 90 seconds after birth, based on the following findings, which newborn does NOT require
resuscitative efforts beyond routine care?
A) HR = 100, RR = 30, peripheral cyanosis, APGAR = 8
B) HR = 158, RR = 40, central cyanosis, APGAR = 6
C) HR = 75, RR = 25, central cyanosis, APGAR = 5
D) HR = 98, RR = 27, peripheral cyanosis, APGAR = 6
ANSWER: A
Diff: 2 Page Ref: 64-65
Standard: Special Patient Populations (Neonatal Care)
Objective: 3
13) You have just assisted in the delivery of a 34-week-gestation infant in the office of a
methadone clinic. After 30 seconds of drying and stimulation, the infant remains limp and cyanotic,
with a pulse of 50. Which of the following is the most appropriate action to take at this point?
A) Begin CPR at a rate of 3 compressions to 1 ventilation.
B) Intubate the trachea, ventilate, reassess, and prepare to transport to a NICU.
C) Begin bag-valve-mask ventilations and establish intravenous access.
D) Continue to warm and dry the infant for an additional 30 seconds, while preparing to intubate.
ANSWER: A
Diff: 3 Page Ref: 60
Standard: Special Patient Populations (Neonatal Care)
TO GET ALL CHAPTERS EMAIL ME AT>>>>>
donc8246@gmail.com
Objective: 5
14) Immediately after delivery, a neonate is crying and moving his arms and legs, but is centrally
cyanotic. You recognize:
A) the need for supplemental oxygen.
B) signs of a normal, healthy infant.
C) the need to prevent further heat loss.
D) signs of a congenital heart abnormality.
ANSWER: B
Diff: 2 Page Ref: 60-61
Standard: Special Patient Populations (Neonatal Care)
Objective: 3
TO GET ALL CHAPTERS EMAIL ME AT>>>>>
donc8246@gmail.com
15) All of the following are accurate methods for obtaining the heart rate of a newborn EXCEPT:
A) a pulse oximeter.
B) 3-lead ECG.
C) palpating a brachial pulse.
D) auscultating heart tones.
ANSWER: C
Diff: 2 Page Ref: 64-65
Standard: Special Patient Populations (Neonatal Care)
Objective: 5
16) You are resuscitating a 15-day-old infant who was found pulseless and apneic in her crib.
Which of the following questions would be most helpful when obtaining a history from the
parents?
A) "Does your daughter have any congenital heart abnormalities?"
B) "Was your baby sleeping face up?"
C) "Is there a family history of heart disease?"
D) "Have any of your other children died from SIDS?"
ANSWER: A
Diff: 3 Page Ref: 76
Standard: Special Patient Populations (Neonatal Care)
Objective: 5
17) You are called to the home of a 72-hour-old infant whose mother is concerned because the
infant has been "vomiting after she eats." Based on this information, which of the following
should you include in your history and physical assessment?
A) End-tidal CO2 monitoring
B) Auscultation of gastric sounds
C) Asking if the baby could have ingested any toxins
D) Assessing the fontanels
ANSWER: D
Diff: 3 Page Ref: 75
Standard: Special Patient Populations (Neonatal Care)
TO GET ALL CHAPTERS EMAIL ME AT>>>>>
donc8246@gmail.com
Objective: 5
TO GET ALL CHAPTERS EMAIL ME AT>>>>>
donc8246@gmail.com
18) You have just assisted in the delivery of an approximately 4 kg newborn whose mother is a
rather petite primapara. On assessment, you note that there is no spontaneous movement of the
infant's right arm at the shoulder, but he moves the elbow and wrist. He otherwise exhibits
vigorous movement and has a 1-minute APGAR score of 9. Which of the following should you
suspect?
A) Neonatal abstinence syndrome
B) Spinal cord damage
C) Fractured clavicle
D) Caput succedaneum
ANSWER: C
Diff: 2 Page Ref: 76
Standard: Special Patient Populations (Neonatal Care) Objective:
6
TO GET ALL CHAPTERS EMAIL ME AT>>>>>
donc8246@gmail.com
19) You are assessing a 1-minute APGAR score on a patient who has peripheral cyanosis and a
heart rate of 98, who cried spontaneously after delivery, is actively moving his extremities, and has
a strong cry. What is the APGAR score of this patient?
A) 9
B) 7
C) 8
D) 10
ANSWER
: C
Diff: 2 Page Ref: 61
Standard: Special Patient Populations (Neonatal Care) Objective:
6
20) A newborn was found pulseless and apneic in his crib. After lengthy attempts at
resuscitation, medical control has advised you to stop resuscitation. When you inform the
parents, they become hysterical and the mother cries, "Give me my baby! I don't want you
touching him!" You should:
A) gently tell the mother that, unfortunately, she cannot hold the baby until the coroner arrives.
B) hand the child to the mother and allow the parents time alone with the child.
C) request that a law enforcement officer remain with the mother.
D) move the infant to the ambulance and transport.
ANSWER: B
Diff: 2 Page Ref: 73
Standard: Special Patient Populations (Neonatal Care) Objective:
5
21) You are called to a residence to care for a 3-hour-old infant in respiratory distress. Your
physical exam reveals retractions, grunting, and tachypnea, along with rhonchi and crackles in the
lungs. The midwife reports that the amniotic fluid had "thin meconium." Which of the following
is the most likely cause of this infant's signs and symptoms?
A) Tetralogy of Fallot
B) Cardiac compromise
C) Persistent fetal circulation
TO GET ALL CHAPTERS EMAIL ME AT>>>>>
donc8246@gmail.com
D) Meconium aspiration

More Related Content

Similar to TEST BANK For Paramedic Care - Principles and Practice, 6th Edition, Volume 1 - 5 by Bledsoe.pdf

OSCE 1 and 2 combined - Dr Ameen Alawadhi
OSCE 1 and 2 combined - Dr Ameen AlawadhiOSCE 1 and 2 combined - Dr Ameen Alawadhi
OSCE 1 and 2 combined - Dr Ameen Alawadhi
askadermatologist
 
Book 2009 krok 2
Book 2009 krok 2Book 2009 krok 2
Book 2009 krok 2
Raj Twix
 
Krok 2 - 2015 Question Paper (General Medicine)
Krok 2 - 2015 Question Paper (General Medicine)Krok 2 - 2015 Question Paper (General Medicine)
Krok 2 - 2015 Question Paper (General Medicine)
Eneutron
 
Case Study Assignment for Unit IIIPurpose The purpose of th.docx
Case Study Assignment for Unit IIIPurpose The purpose of th.docxCase Study Assignment for Unit IIIPurpose The purpose of th.docx
Case Study Assignment for Unit IIIPurpose The purpose of th.docx
wendolynhalbert
 
Fundamentals of nursing practice exam
Fundamentals of nursing practice examFundamentals of nursing practice exam
Fundamentals of nursing practice exam
Nursing Path
 
Book 2011 krok 2
Book 2011 krok 2Book 2011 krok 2
Book 2011 krok 2
Raj Twix
 
Krok 2 - 2010 Question Paper (General Medicine)
Krok 2 - 2010 Question Paper (General Medicine)Krok 2 - 2010 Question Paper (General Medicine)
Krok 2 - 2010 Question Paper (General Medicine)
Eneutron
 
ANTI TOBACCO DAY essay.docx
ANTI TOBACCO DAY essay.docxANTI TOBACCO DAY essay.docx
ANTI TOBACCO DAY essay.docx
anjalatchi
 
Set 1.pptx
Set 1.pptxSet 1.pptx
Set 1.pptx
NishaGhimire2
 
TEST BANK FOR Anatomy & PhysiologyHealth Professions, An Interactive Journey,...
TEST BANK FOR Anatomy & PhysiologyHealth Professions, An Interactive Journey,...TEST BANK FOR Anatomy & PhysiologyHealth Professions, An Interactive Journey,...
TEST BANK FOR Anatomy & PhysiologyHealth Professions, An Interactive Journey,...
rightmanforbloodline
 
Mcqs general epidemiology
Mcqs   general epidemiologyMcqs   general epidemiology
Mcqs general epidemiology
monaaboserea
 
MCQS_FOR_Obstetrics_and_Gynaecology.pdf
MCQS_FOR_Obstetrics_and_Gynaecology.pdfMCQS_FOR_Obstetrics_and_Gynaecology.pdf
MCQS_FOR_Obstetrics_and_Gynaecology.pdf
Ankur Shah
 
Clinical Cases Study for Intra-abdominal infections
Clinical Cases Study for Intra-abdominal infections Clinical Cases Study for Intra-abdominal infections
Clinical Cases Study for Intra-abdominal infections
Sameh Abdel-ghany
 
Drs. Olson’s and Jackson’s CMC Pediatric X-Ray Mastery: August Cases
Drs. Olson’s and Jackson’s CMC Pediatric X-Ray Mastery: August CasesDrs. Olson’s and Jackson’s CMC Pediatric X-Ray Mastery: August Cases
Drs. Olson’s and Jackson’s CMC Pediatric X-Ray Mastery: August Cases
Sean M. Fox
 
Usmle step 2 ck september 2020 recalls (dr notes.com)
Usmle step 2 ck september 2020 recalls (dr notes.com)Usmle step 2 ck september 2020 recalls (dr notes.com)
Usmle step 2 ck september 2020 recalls (dr notes.com)
usmlematerialsnet
 
Krok 2 - 2014 Question Paper (General Medicine)
Krok 2 - 2014 Question Paper (General Medicine)Krok 2 - 2014 Question Paper (General Medicine)
Krok 2 - 2014 Question Paper (General Medicine)
Eneutron
 
Krok 2 - 2007 Question Paper (General Medicine)
Krok 2 - 2007 Question Paper (General Medicine)Krok 2 - 2007 Question Paper (General Medicine)
Krok 2 - 2007 Question Paper (General Medicine)
Eneutron
 

Similar to TEST BANK For Paramedic Care - Principles and Practice, 6th Edition, Volume 1 - 5 by Bledsoe.pdf (20)

OSCE 1 and 2 combined - Dr Ameen Alawadhi
OSCE 1 and 2 combined - Dr Ameen AlawadhiOSCE 1 and 2 combined - Dr Ameen Alawadhi
OSCE 1 and 2 combined - Dr Ameen Alawadhi
 
Book 2009 krok 2
Book 2009 krok 2Book 2009 krok 2
Book 2009 krok 2
 
Krok 2 - 2015 Question Paper (General Medicine)
Krok 2 - 2015 Question Paper (General Medicine)Krok 2 - 2015 Question Paper (General Medicine)
Krok 2 - 2015 Question Paper (General Medicine)
 
Case Study Assignment for Unit IIIPurpose The purpose of th.docx
Case Study Assignment for Unit IIIPurpose The purpose of th.docxCase Study Assignment for Unit IIIPurpose The purpose of th.docx
Case Study Assignment for Unit IIIPurpose The purpose of th.docx
 
Document
DocumentDocument
Document
 
Fundamentals of nursing practice exam
Fundamentals of nursing practice examFundamentals of nursing practice exam
Fundamentals of nursing practice exam
 
Book 2011 krok 2
Book 2011 krok 2Book 2011 krok 2
Book 2011 krok 2
 
Clinical cases (2) june 9, 13
Clinical cases  (2) june 9, 13Clinical cases  (2) june 9, 13
Clinical cases (2) june 9, 13
 
Krok 2 - 2010 Question Paper (General Medicine)
Krok 2 - 2010 Question Paper (General Medicine)Krok 2 - 2010 Question Paper (General Medicine)
Krok 2 - 2010 Question Paper (General Medicine)
 
Clinical cases (1) june 16, 13
Clinical cases  (1) june 16, 13Clinical cases  (1) june 16, 13
Clinical cases (1) june 16, 13
 
ANTI TOBACCO DAY essay.docx
ANTI TOBACCO DAY essay.docxANTI TOBACCO DAY essay.docx
ANTI TOBACCO DAY essay.docx
 
Set 1.pptx
Set 1.pptxSet 1.pptx
Set 1.pptx
 
TEST BANK FOR Anatomy & PhysiologyHealth Professions, An Interactive Journey,...
TEST BANK FOR Anatomy & PhysiologyHealth Professions, An Interactive Journey,...TEST BANK FOR Anatomy & PhysiologyHealth Professions, An Interactive Journey,...
TEST BANK FOR Anatomy & PhysiologyHealth Professions, An Interactive Journey,...
 
Mcqs general epidemiology
Mcqs   general epidemiologyMcqs   general epidemiology
Mcqs general epidemiology
 
MCQS_FOR_Obstetrics_and_Gynaecology.pdf
MCQS_FOR_Obstetrics_and_Gynaecology.pdfMCQS_FOR_Obstetrics_and_Gynaecology.pdf
MCQS_FOR_Obstetrics_and_Gynaecology.pdf
 
Clinical Cases Study for Intra-abdominal infections
Clinical Cases Study for Intra-abdominal infections Clinical Cases Study for Intra-abdominal infections
Clinical Cases Study for Intra-abdominal infections
 
Drs. Olson’s and Jackson’s CMC Pediatric X-Ray Mastery: August Cases
Drs. Olson’s and Jackson’s CMC Pediatric X-Ray Mastery: August CasesDrs. Olson’s and Jackson’s CMC Pediatric X-Ray Mastery: August Cases
Drs. Olson’s and Jackson’s CMC Pediatric X-Ray Mastery: August Cases
 
Usmle step 2 ck september 2020 recalls (dr notes.com)
Usmle step 2 ck september 2020 recalls (dr notes.com)Usmle step 2 ck september 2020 recalls (dr notes.com)
Usmle step 2 ck september 2020 recalls (dr notes.com)
 
Krok 2 - 2014 Question Paper (General Medicine)
Krok 2 - 2014 Question Paper (General Medicine)Krok 2 - 2014 Question Paper (General Medicine)
Krok 2 - 2014 Question Paper (General Medicine)
 
Krok 2 - 2007 Question Paper (General Medicine)
Krok 2 - 2007 Question Paper (General Medicine)Krok 2 - 2007 Question Paper (General Medicine)
Krok 2 - 2007 Question Paper (General Medicine)
 

More from Donc Test

Libby/Libby/Hodge Financial Accounting 11th Edition TEST BANK & SOLUTION MANU...
Libby/Libby/Hodge Financial Accounting 11th Edition TEST BANK & SOLUTION MANU...Libby/Libby/Hodge Financial Accounting 11th Edition TEST BANK & SOLUTION MANU...
Libby/Libby/Hodge Financial Accounting 11th Edition TEST BANK & SOLUTION MANU...
Donc Test
 
Solution Manual For Financial Accounting, 8th Canadian Edition 2024, by Libby...
Solution Manual For Financial Accounting, 8th Canadian Edition 2024, by Libby...Solution Manual For Financial Accounting, 8th Canadian Edition 2024, by Libby...
Solution Manual For Financial Accounting, 8th Canadian Edition 2024, by Libby...
Donc Test
 
TEST BANK For Community and Public Health Nursing: Evidence for Practice, 4th...
TEST BANK For Community and Public Health Nursing: Evidence for Practice, 4th...TEST BANK For Community and Public Health Nursing: Evidence for Practice, 4th...
TEST BANK For Community and Public Health Nursing: Evidence for Practice, 4th...
Donc Test
 
TEST BANK For Community and Public Health Nursing: Evidence for Practice, 3rd...
TEST BANK For Community and Public Health Nursing: Evidence for Practice, 3rd...TEST BANK For Community and Public Health Nursing: Evidence for Practice, 3rd...
TEST BANK For Community and Public Health Nursing: Evidence for Practice, 3rd...
Donc Test
 
TEST BANK For Community Health Nursing A Canadian Perspective, 5th Edition by...
TEST BANK For Community Health Nursing A Canadian Perspective, 5th Edition by...TEST BANK For Community Health Nursing A Canadian Perspective, 5th Edition by...
TEST BANK For Community Health Nursing A Canadian Perspective, 5th Edition by...
Donc Test
 
TEST BANK For CURRENT Diagnosis & Treatment Pediatrics, 26th Edition by Maya ...
TEST BANK For CURRENT Diagnosis & Treatment Pediatrics, 26th Edition by Maya ...TEST BANK For CURRENT Diagnosis & Treatment Pediatrics, 26th Edition by Maya ...
TEST BANK For CURRENT Diagnosis & Treatment Pediatrics, 26th Edition by Maya ...
Donc Test
 
TEST BANK For Current Medical Diagnosis And Treatment 2024, 63rd Edition By M...
TEST BANK For Current Medical Diagnosis And Treatment 2024, 63rd Edition By M...TEST BANK For Current Medical Diagnosis And Treatment 2024, 63rd Edition By M...
TEST BANK For Current Medical Diagnosis And Treatment 2024, 63rd Edition By M...
Donc Test
 
TEST BANK For Davis Advantage for Pathophysiology Introductory Concepts and C...
TEST BANK For Davis Advantage for Pathophysiology Introductory Concepts and C...TEST BANK For Davis Advantage for Pathophysiology Introductory Concepts and C...
TEST BANK For Davis Advantage for Pathophysiology Introductory Concepts and C...
Donc Test
 
TEST BANK For Discovering the Life Span, 5th Edition Robert S. Feldman, Verif...
TEST BANK For Discovering the Life Span, 5th Edition Robert S. Feldman, Verif...TEST BANK For Discovering the Life Span, 5th Edition Robert S. Feldman, Verif...
TEST BANK For Discovering the Life Span, 5th Edition Robert S. Feldman, Verif...
Donc Test
 
Test Bank - Karch Focus on Nursing Pharmacology 9th Edition by Rebecca Tucker...
Test Bank - Karch Focus on Nursing Pharmacology 9th Edition by Rebecca Tucker...Test Bank - Karch Focus on Nursing Pharmacology 9th Edition by Rebecca Tucker...
Test Bank - Karch Focus on Nursing Pharmacology 9th Edition by Rebecca Tucker...
Donc Test
 
Test Bank For Clinical Nursing Skills and Techniques 10th Edition (1).pdf
Test Bank For Clinical Nursing Skills and Techniques 10th Edition (1).pdfTest Bank For Clinical Nursing Skills and Techniques 10th Edition (1).pdf
Test Bank For Clinical Nursing Skills and Techniques 10th Edition (1).pdf
Donc Test
 
Test Bank for Anatomy of Oriented Structure 8th edition.pdf
Test Bank for Anatomy of Oriented Structure 8th edition.pdfTest Bank for Anatomy of Oriented Structure 8th edition.pdf
Test Bank for Anatomy of Oriented Structure 8th edition.pdf
Donc Test
 
TEST BANK Essentials of dental radiography 9th edition by Evelyn Thomson, Orl...
TEST BANK Essentials of dental radiography 9th edition by Evelyn Thomson, Orl...TEST BANK Essentials of dental radiography 9th edition by Evelyn Thomson, Orl...
TEST BANK Essentials of dental radiography 9th edition by Evelyn Thomson, Orl...
Donc Test
 
Test bank clinical nursing skills a concept based approach 4e pearson educati...
Test bank clinical nursing skills a concept based approach 4e pearson educati...Test bank clinical nursing skills a concept based approach 4e pearson educati...
Test bank clinical nursing skills a concept based approach 4e pearson educati...
Donc Test
 
TEST BANK For, Information Technology Project Management 9th Edition Kathy Sc...
TEST BANK For, Information Technology Project Management 9th Edition Kathy Sc...TEST BANK For, Information Technology Project Management 9th Edition Kathy Sc...
TEST BANK For, Information Technology Project Management 9th Edition Kathy Sc...
Donc Test
 
TEST BANK For Wong's Nursing Care of Infants and Children, 11th Edition by Ma...
TEST BANK For Wong's Nursing Care of Infants and Children, 11th Edition by Ma...TEST BANK For Wong's Nursing Care of Infants and Children, 11th Edition by Ma...
TEST BANK For Wong's Nursing Care of Infants and Children, 11th Edition by Ma...
Donc Test
 
TEST BANK for Wilkins’ Clinical Assessment in Respiratory Care, 9th Edition b...
TEST BANK for Wilkins’ Clinical Assessment in Respiratory Care, 9th Edition b...TEST BANK for Wilkins’ Clinical Assessment in Respiratory Care, 9th Edition b...
TEST BANK for Wilkins’ Clinical Assessment in Respiratory Care, 9th Edition b...
Donc Test
 
TEST BANK For Understanding the Essentials of Critical Care Nursing, 3rd Edit...
TEST BANK For Understanding the Essentials of Critical Care Nursing, 3rd Edit...TEST BANK For Understanding the Essentials of Critical Care Nursing, 3rd Edit...
TEST BANK For Understanding the Essentials of Critical Care Nursing, 3rd Edit...
Donc Test
 
TEST BANK For Timby's Introductory Medical-Surgical Nursing, 13th Edition by ...
TEST BANK For Timby's Introductory Medical-Surgical Nursing, 13th Edition by ...TEST BANK For Timby's Introductory Medical-Surgical Nursing, 13th Edition by ...
TEST BANK For Timby's Introductory Medical-Surgical Nursing, 13th Edition by ...
Donc Test
 
TEST BANK for The Nursing Assistant Acute, Subacute, and Long-Term Care, 6th ...
TEST BANK for The Nursing Assistant Acute, Subacute, and Long-Term Care, 6th ...TEST BANK for The Nursing Assistant Acute, Subacute, and Long-Term Care, 6th ...
TEST BANK for The Nursing Assistant Acute, Subacute, and Long-Term Care, 6th ...
Donc Test
 

More from Donc Test (20)

Libby/Libby/Hodge Financial Accounting 11th Edition TEST BANK & SOLUTION MANU...
Libby/Libby/Hodge Financial Accounting 11th Edition TEST BANK & SOLUTION MANU...Libby/Libby/Hodge Financial Accounting 11th Edition TEST BANK & SOLUTION MANU...
Libby/Libby/Hodge Financial Accounting 11th Edition TEST BANK & SOLUTION MANU...
 
Solution Manual For Financial Accounting, 8th Canadian Edition 2024, by Libby...
Solution Manual For Financial Accounting, 8th Canadian Edition 2024, by Libby...Solution Manual For Financial Accounting, 8th Canadian Edition 2024, by Libby...
Solution Manual For Financial Accounting, 8th Canadian Edition 2024, by Libby...
 
TEST BANK For Community and Public Health Nursing: Evidence for Practice, 4th...
TEST BANK For Community and Public Health Nursing: Evidence for Practice, 4th...TEST BANK For Community and Public Health Nursing: Evidence for Practice, 4th...
TEST BANK For Community and Public Health Nursing: Evidence for Practice, 4th...
 
TEST BANK For Community and Public Health Nursing: Evidence for Practice, 3rd...
TEST BANK For Community and Public Health Nursing: Evidence for Practice, 3rd...TEST BANK For Community and Public Health Nursing: Evidence for Practice, 3rd...
TEST BANK For Community and Public Health Nursing: Evidence for Practice, 3rd...
 
TEST BANK For Community Health Nursing A Canadian Perspective, 5th Edition by...
TEST BANK For Community Health Nursing A Canadian Perspective, 5th Edition by...TEST BANK For Community Health Nursing A Canadian Perspective, 5th Edition by...
TEST BANK For Community Health Nursing A Canadian Perspective, 5th Edition by...
 
TEST BANK For CURRENT Diagnosis & Treatment Pediatrics, 26th Edition by Maya ...
TEST BANK For CURRENT Diagnosis & Treatment Pediatrics, 26th Edition by Maya ...TEST BANK For CURRENT Diagnosis & Treatment Pediatrics, 26th Edition by Maya ...
TEST BANK For CURRENT Diagnosis & Treatment Pediatrics, 26th Edition by Maya ...
 
TEST BANK For Current Medical Diagnosis And Treatment 2024, 63rd Edition By M...
TEST BANK For Current Medical Diagnosis And Treatment 2024, 63rd Edition By M...TEST BANK For Current Medical Diagnosis And Treatment 2024, 63rd Edition By M...
TEST BANK For Current Medical Diagnosis And Treatment 2024, 63rd Edition By M...
 
TEST BANK For Davis Advantage for Pathophysiology Introductory Concepts and C...
TEST BANK For Davis Advantage for Pathophysiology Introductory Concepts and C...TEST BANK For Davis Advantage for Pathophysiology Introductory Concepts and C...
TEST BANK For Davis Advantage for Pathophysiology Introductory Concepts and C...
 
TEST BANK For Discovering the Life Span, 5th Edition Robert S. Feldman, Verif...
TEST BANK For Discovering the Life Span, 5th Edition Robert S. Feldman, Verif...TEST BANK For Discovering the Life Span, 5th Edition Robert S. Feldman, Verif...
TEST BANK For Discovering the Life Span, 5th Edition Robert S. Feldman, Verif...
 
Test Bank - Karch Focus on Nursing Pharmacology 9th Edition by Rebecca Tucker...
Test Bank - Karch Focus on Nursing Pharmacology 9th Edition by Rebecca Tucker...Test Bank - Karch Focus on Nursing Pharmacology 9th Edition by Rebecca Tucker...
Test Bank - Karch Focus on Nursing Pharmacology 9th Edition by Rebecca Tucker...
 
Test Bank For Clinical Nursing Skills and Techniques 10th Edition (1).pdf
Test Bank For Clinical Nursing Skills and Techniques 10th Edition (1).pdfTest Bank For Clinical Nursing Skills and Techniques 10th Edition (1).pdf
Test Bank For Clinical Nursing Skills and Techniques 10th Edition (1).pdf
 
Test Bank for Anatomy of Oriented Structure 8th edition.pdf
Test Bank for Anatomy of Oriented Structure 8th edition.pdfTest Bank for Anatomy of Oriented Structure 8th edition.pdf
Test Bank for Anatomy of Oriented Structure 8th edition.pdf
 
TEST BANK Essentials of dental radiography 9th edition by Evelyn Thomson, Orl...
TEST BANK Essentials of dental radiography 9th edition by Evelyn Thomson, Orl...TEST BANK Essentials of dental radiography 9th edition by Evelyn Thomson, Orl...
TEST BANK Essentials of dental radiography 9th edition by Evelyn Thomson, Orl...
 
Test bank clinical nursing skills a concept based approach 4e pearson educati...
Test bank clinical nursing skills a concept based approach 4e pearson educati...Test bank clinical nursing skills a concept based approach 4e pearson educati...
Test bank clinical nursing skills a concept based approach 4e pearson educati...
 
TEST BANK For, Information Technology Project Management 9th Edition Kathy Sc...
TEST BANK For, Information Technology Project Management 9th Edition Kathy Sc...TEST BANK For, Information Technology Project Management 9th Edition Kathy Sc...
TEST BANK For, Information Technology Project Management 9th Edition Kathy Sc...
 
TEST BANK For Wong's Nursing Care of Infants and Children, 11th Edition by Ma...
TEST BANK For Wong's Nursing Care of Infants and Children, 11th Edition by Ma...TEST BANK For Wong's Nursing Care of Infants and Children, 11th Edition by Ma...
TEST BANK For Wong's Nursing Care of Infants and Children, 11th Edition by Ma...
 
TEST BANK for Wilkins’ Clinical Assessment in Respiratory Care, 9th Edition b...
TEST BANK for Wilkins’ Clinical Assessment in Respiratory Care, 9th Edition b...TEST BANK for Wilkins’ Clinical Assessment in Respiratory Care, 9th Edition b...
TEST BANK for Wilkins’ Clinical Assessment in Respiratory Care, 9th Edition b...
 
TEST BANK For Understanding the Essentials of Critical Care Nursing, 3rd Edit...
TEST BANK For Understanding the Essentials of Critical Care Nursing, 3rd Edit...TEST BANK For Understanding the Essentials of Critical Care Nursing, 3rd Edit...
TEST BANK For Understanding the Essentials of Critical Care Nursing, 3rd Edit...
 
TEST BANK For Timby's Introductory Medical-Surgical Nursing, 13th Edition by ...
TEST BANK For Timby's Introductory Medical-Surgical Nursing, 13th Edition by ...TEST BANK For Timby's Introductory Medical-Surgical Nursing, 13th Edition by ...
TEST BANK For Timby's Introductory Medical-Surgical Nursing, 13th Edition by ...
 
TEST BANK for The Nursing Assistant Acute, Subacute, and Long-Term Care, 6th ...
TEST BANK for The Nursing Assistant Acute, Subacute, and Long-Term Care, 6th ...TEST BANK for The Nursing Assistant Acute, Subacute, and Long-Term Care, 6th ...
TEST BANK for The Nursing Assistant Acute, Subacute, and Long-Term Care, 6th ...
 

Recently uploaded

CDSCO and Phamacovigilance {Regulatory body in India}
CDSCO and Phamacovigilance {Regulatory body in India}CDSCO and Phamacovigilance {Regulatory body in India}
CDSCO and Phamacovigilance {Regulatory body in India}
NEHA GUPTA
 
Ocular injury ppt Upendra pal optometrist upums saifai etawah
Ocular injury  ppt  Upendra pal  optometrist upums saifai etawahOcular injury  ppt  Upendra pal  optometrist upums saifai etawah
Ocular injury ppt Upendra pal optometrist upums saifai etawah
pal078100
 
Hemodialysis: Chapter 3, Dialysis Water Unit - Dr.Gawad
Hemodialysis: Chapter 3, Dialysis Water Unit - Dr.GawadHemodialysis: Chapter 3, Dialysis Water Unit - Dr.Gawad
Hemodialysis: Chapter 3, Dialysis Water Unit - Dr.Gawad
NephroTube - Dr.Gawad
 
ARTHROLOGY PPT NCISM SYLLABUS AYURVEDA STUDENTS
ARTHROLOGY PPT NCISM SYLLABUS AYURVEDA STUDENTSARTHROLOGY PPT NCISM SYLLABUS AYURVEDA STUDENTS
ARTHROLOGY PPT NCISM SYLLABUS AYURVEDA STUDENTS
Dr. Vinay Pareek
 
Triangles of Neck and Clinical Correlation by Dr. RIG.pptx
Triangles of Neck and Clinical Correlation by Dr. RIG.pptxTriangles of Neck and Clinical Correlation by Dr. RIG.pptx
Triangles of Neck and Clinical Correlation by Dr. RIG.pptx
Dr. Rabia Inam Gandapore
 
Maxilla, Mandible & Hyoid Bone & Clinical Correlations by Dr. RIG.pptx
Maxilla, Mandible & Hyoid Bone & Clinical Correlations by Dr. RIG.pptxMaxilla, Mandible & Hyoid Bone & Clinical Correlations by Dr. RIG.pptx
Maxilla, Mandible & Hyoid Bone & Clinical Correlations by Dr. RIG.pptx
Dr. Rabia Inam Gandapore
 
How STIs Influence the Development of Pelvic Inflammatory Disease.pptx
How STIs Influence the Development of Pelvic Inflammatory Disease.pptxHow STIs Influence the Development of Pelvic Inflammatory Disease.pptx
How STIs Influence the Development of Pelvic Inflammatory Disease.pptx
FFragrant
 
SURGICAL ANATOMY OF THE RETROPERITONEUM, ADRENALS, KIDNEYS AND URETERS.pptx
SURGICAL ANATOMY OF THE RETROPERITONEUM, ADRENALS, KIDNEYS AND URETERS.pptxSURGICAL ANATOMY OF THE RETROPERITONEUM, ADRENALS, KIDNEYS AND URETERS.pptx
SURGICAL ANATOMY OF THE RETROPERITONEUM, ADRENALS, KIDNEYS AND URETERS.pptx
Bright Chipili
 
A Classical Text Review on Basavarajeeyam
A Classical Text Review on BasavarajeeyamA Classical Text Review on Basavarajeeyam
A Classical Text Review on Basavarajeeyam
Dr. Jyothirmai Paindla
 
Best Ayurvedic medicine for Gas and Indigestion
Best Ayurvedic medicine for Gas and IndigestionBest Ayurvedic medicine for Gas and Indigestion
Best Ayurvedic medicine for Gas and Indigestion
SwastikAyurveda
 
Cervical & Brachial Plexus By Dr. RIG.pptx
Cervical & Brachial Plexus By Dr. RIG.pptxCervical & Brachial Plexus By Dr. RIG.pptx
Cervical & Brachial Plexus By Dr. RIG.pptx
Dr. Rabia Inam Gandapore
 
KDIGO 2024 guidelines for diabetologists
KDIGO 2024 guidelines for diabetologistsKDIGO 2024 guidelines for diabetologists
KDIGO 2024 guidelines for diabetologists
د.محمود نجيب
 
Ozempic: Preoperative Management of Patients on GLP-1 Receptor Agonists
Ozempic: Preoperative Management of Patients on GLP-1 Receptor Agonists  Ozempic: Preoperative Management of Patients on GLP-1 Receptor Agonists
Ozempic: Preoperative Management of Patients on GLP-1 Receptor Agonists
Saeid Safari
 
Sex determination from mandible pelvis and skull
Sex determination from mandible pelvis and skullSex determination from mandible pelvis and skull
Sex determination from mandible pelvis and skull
ShashankRoodkee
 
Superficial & Deep Fascia of the NECK.pptx
Superficial & Deep Fascia of the NECK.pptxSuperficial & Deep Fascia of the NECK.pptx
Superficial & Deep Fascia of the NECK.pptx
Dr. Rabia Inam Gandapore
 
Aortic Association CBL Pilot April 19 – 20 Bern
Aortic Association CBL Pilot April 19 – 20 BernAortic Association CBL Pilot April 19 – 20 Bern
Aortic Association CBL Pilot April 19 – 20 Bern
suvadeepdas911
 
Physiology of Special Chemical Sensation of Taste
Physiology of Special Chemical Sensation of TastePhysiology of Special Chemical Sensation of Taste
Physiology of Special Chemical Sensation of Taste
MedicoseAcademics
 
Adv. biopharm. APPLICATION OF PHARMACOKINETICS : TARGETED DRUG DELIVERY SYSTEMS
Adv. biopharm. APPLICATION OF PHARMACOKINETICS : TARGETED DRUG DELIVERY SYSTEMSAdv. biopharm. APPLICATION OF PHARMACOKINETICS : TARGETED DRUG DELIVERY SYSTEMS
Adv. biopharm. APPLICATION OF PHARMACOKINETICS : TARGETED DRUG DELIVERY SYSTEMS
AkankshaAshtankar
 
Cardiac Assessment for B.sc Nursing Student.pdf
Cardiac Assessment for B.sc Nursing Student.pdfCardiac Assessment for B.sc Nursing Student.pdf
Cardiac Assessment for B.sc Nursing Student.pdf
shivalingatalekar1
 
Flu Vaccine Alert in Bangalore Karnataka
Flu Vaccine Alert in Bangalore KarnatakaFlu Vaccine Alert in Bangalore Karnataka
Flu Vaccine Alert in Bangalore Karnataka
addon Scans
 

Recently uploaded (20)

CDSCO and Phamacovigilance {Regulatory body in India}
CDSCO and Phamacovigilance {Regulatory body in India}CDSCO and Phamacovigilance {Regulatory body in India}
CDSCO and Phamacovigilance {Regulatory body in India}
 
Ocular injury ppt Upendra pal optometrist upums saifai etawah
Ocular injury  ppt  Upendra pal  optometrist upums saifai etawahOcular injury  ppt  Upendra pal  optometrist upums saifai etawah
Ocular injury ppt Upendra pal optometrist upums saifai etawah
 
Hemodialysis: Chapter 3, Dialysis Water Unit - Dr.Gawad
Hemodialysis: Chapter 3, Dialysis Water Unit - Dr.GawadHemodialysis: Chapter 3, Dialysis Water Unit - Dr.Gawad
Hemodialysis: Chapter 3, Dialysis Water Unit - Dr.Gawad
 
ARTHROLOGY PPT NCISM SYLLABUS AYURVEDA STUDENTS
ARTHROLOGY PPT NCISM SYLLABUS AYURVEDA STUDENTSARTHROLOGY PPT NCISM SYLLABUS AYURVEDA STUDENTS
ARTHROLOGY PPT NCISM SYLLABUS AYURVEDA STUDENTS
 
Triangles of Neck and Clinical Correlation by Dr. RIG.pptx
Triangles of Neck and Clinical Correlation by Dr. RIG.pptxTriangles of Neck and Clinical Correlation by Dr. RIG.pptx
Triangles of Neck and Clinical Correlation by Dr. RIG.pptx
 
Maxilla, Mandible & Hyoid Bone & Clinical Correlations by Dr. RIG.pptx
Maxilla, Mandible & Hyoid Bone & Clinical Correlations by Dr. RIG.pptxMaxilla, Mandible & Hyoid Bone & Clinical Correlations by Dr. RIG.pptx
Maxilla, Mandible & Hyoid Bone & Clinical Correlations by Dr. RIG.pptx
 
How STIs Influence the Development of Pelvic Inflammatory Disease.pptx
How STIs Influence the Development of Pelvic Inflammatory Disease.pptxHow STIs Influence the Development of Pelvic Inflammatory Disease.pptx
How STIs Influence the Development of Pelvic Inflammatory Disease.pptx
 
SURGICAL ANATOMY OF THE RETROPERITONEUM, ADRENALS, KIDNEYS AND URETERS.pptx
SURGICAL ANATOMY OF THE RETROPERITONEUM, ADRENALS, KIDNEYS AND URETERS.pptxSURGICAL ANATOMY OF THE RETROPERITONEUM, ADRENALS, KIDNEYS AND URETERS.pptx
SURGICAL ANATOMY OF THE RETROPERITONEUM, ADRENALS, KIDNEYS AND URETERS.pptx
 
A Classical Text Review on Basavarajeeyam
A Classical Text Review on BasavarajeeyamA Classical Text Review on Basavarajeeyam
A Classical Text Review on Basavarajeeyam
 
Best Ayurvedic medicine for Gas and Indigestion
Best Ayurvedic medicine for Gas and IndigestionBest Ayurvedic medicine for Gas and Indigestion
Best Ayurvedic medicine for Gas and Indigestion
 
Cervical & Brachial Plexus By Dr. RIG.pptx
Cervical & Brachial Plexus By Dr. RIG.pptxCervical & Brachial Plexus By Dr. RIG.pptx
Cervical & Brachial Plexus By Dr. RIG.pptx
 
KDIGO 2024 guidelines for diabetologists
KDIGO 2024 guidelines for diabetologistsKDIGO 2024 guidelines for diabetologists
KDIGO 2024 guidelines for diabetologists
 
Ozempic: Preoperative Management of Patients on GLP-1 Receptor Agonists
Ozempic: Preoperative Management of Patients on GLP-1 Receptor Agonists  Ozempic: Preoperative Management of Patients on GLP-1 Receptor Agonists
Ozempic: Preoperative Management of Patients on GLP-1 Receptor Agonists
 
Sex determination from mandible pelvis and skull
Sex determination from mandible pelvis and skullSex determination from mandible pelvis and skull
Sex determination from mandible pelvis and skull
 
Superficial & Deep Fascia of the NECK.pptx
Superficial & Deep Fascia of the NECK.pptxSuperficial & Deep Fascia of the NECK.pptx
Superficial & Deep Fascia of the NECK.pptx
 
Aortic Association CBL Pilot April 19 – 20 Bern
Aortic Association CBL Pilot April 19 – 20 BernAortic Association CBL Pilot April 19 – 20 Bern
Aortic Association CBL Pilot April 19 – 20 Bern
 
Physiology of Special Chemical Sensation of Taste
Physiology of Special Chemical Sensation of TastePhysiology of Special Chemical Sensation of Taste
Physiology of Special Chemical Sensation of Taste
 
Adv. biopharm. APPLICATION OF PHARMACOKINETICS : TARGETED DRUG DELIVERY SYSTEMS
Adv. biopharm. APPLICATION OF PHARMACOKINETICS : TARGETED DRUG DELIVERY SYSTEMSAdv. biopharm. APPLICATION OF PHARMACOKINETICS : TARGETED DRUG DELIVERY SYSTEMS
Adv. biopharm. APPLICATION OF PHARMACOKINETICS : TARGETED DRUG DELIVERY SYSTEMS
 
Cardiac Assessment for B.sc Nursing Student.pdf
Cardiac Assessment for B.sc Nursing Student.pdfCardiac Assessment for B.sc Nursing Student.pdf
Cardiac Assessment for B.sc Nursing Student.pdf
 
Flu Vaccine Alert in Bangalore Karnataka
Flu Vaccine Alert in Bangalore KarnatakaFlu Vaccine Alert in Bangalore Karnataka
Flu Vaccine Alert in Bangalore Karnataka
 

TEST BANK For Paramedic Care - Principles and Practice, 6th Edition, Volume 1 - 5 by Bledsoe.pdf

  • 1. TO GET ALL CHAPTERS EMAIL ME AT>>>>> donc8246@gmail.com TEST BANK Paramedic Care- Principles & Practice, 6th Edition, Volume 5 by Bledsoe, Chapters 1 - 16
  • 2. TO GET ALL CHAPTERS EMAIL ME AT>>>>> donc8246@gmail.com Test Bank Paramedic Care: Principles & Practice V.5, 6e (Bledsoe) Volume 5: Special Considerations and Operations Table of contents Chapter 1. Gynecology Chapter 2. Obstetrics Chapter 3. Neonatology Chapter 4. Pediatrics Chapter 5. Geriatrics Chapter 6. Abuse, Neglect, and Assault Chapter 7. The Challenged Patient Chapter 8. Acute Interventions for the Chronic Care Patient Chapter 9. Ground Ambulance Operations Chapter 10. Air Medical Operations Chapter 11. Multiple-Casualty Incidents and Incident Management Chapter 12. Rescue Awareness and Operations Chapter 13. Hazardous Materials Chapter 14. Crime Scene Awareness Chapter 15. Rural EMS Chapter 16. Responding to Terrorist Acts
  • 3. TO GET ALL CHAPTERS EMAIL ME AT>>>>> donc8246@gmail.com Paramedic Care: Principles & Practice V.5, 6e (Bledsoe) Volume 5: Special Considerations and Operations Chapter 1: Gynecology 1) The paramedic accurately describes the difference between endometritis and endometriosis when she states: A) "Endometriosis is an infection of the uterine lining, while endometritis occurs when endometrial tissue is found outside the uterus." B) "Endometriosis occurs in women under age 40, while endometriosis is more common in women who are older." C) "Endometritis is an infection of the uterine lining, while endometriosis occurs when endometrial tissue is found outside the uterus." D) "Endometritis patients can be transported code 2, while those with endometriosis should always go code 3." ANSWER: C Diff: 2 Page Ref: 12 Standard: Medicine (Gynecology) Objective: 1 2) Your patient tells you that she is being treated for cystitis. You recognize that she is being treated for: A) ovarian cysts. B) pelvic inflammatory disease. C) a urinary tract infection. D) an ectopic pregnancy. ANSWER: C Diff: 1 Page Ref: 12 Standard: Medicine (Gynecology) Objective: 1 3) Which of the following is TRUE of mittelschmerz?
  • 4. TO GET ALL CHAPTERS EMAIL ME AT>>>>> donc8246@gmail.com A) It is typically located unilaterally in one of the upper abdominal quadrants. B) It is usually accompanied by heavy vaginal bleeding.
  • 5. TO GET ALL CHAPTERS EMAIL ME AT>>>>> donc8246@gmail.com C) It is associated with ovulation. D) It is a sign of ectopic pregnancy. ANSWER: C Diff: 1 Page Ref: 12 Standard: Medicine (Gynecology) Objective: 1
  • 6. TO GET ALL CHAPTERS EMAIL ME AT>>>>> donc8246@gmail.com 4) Menorrhagia is: A) irregular cycles of menstruation. B) painful menstruation. C) absence of menstruation. D) excessive menstrual flow. ANSWER: D Diff: 1 Page Ref: 13 Standard: Medicine (Gynecology) Objective: 1 5) Mittelschmerz refers to which of the following? A) Purulent vaginal discharge B) False labor pains C) Midcycle abdominal pain D) Painful urination ANSWER: C Diff: 1 Page Ref: 12 Standard: Medicine (Gynecology) Objective: 1 6) Which of the following occurs during the proliferative phase of the menstrual cycle? A) Ovulation B) An increase in uterine vascularity C) Endometrial thickening D) A drop in estrogen levels ANSWER: C Diff: 1 Page Ref: 6 Standard: Medicine (Gynecology) Objective: 2 7) The innermost lining of the uterus is called the: A) myometrium. B) perimetrium.
  • 7. TO GET ALL CHAPTERS EMAIL ME AT>>>>> donc8246@gmail.com C) endometrium
  • 8. TO GET ALL CHAPTERS EMAIL ME AT>>>>> donc8246@gmail.com D) vasometrium. ANSWER: C Diff: 1 Page Ref: 5 Standard: Medicine (Gynecology) Objective: 2
  • 9. TO GET ALL CHAPTERS EMAIL ME AT>>>>> donc8246@gmail.com 8) A 22-year-old woman presents in moderate distress, complaining of diffuse lower abdominal pain. She states that the pain has become progressively worse for the past two weeks and she is now unable to walk without an increase in pain. Your physical exam reveals severe pain with palpation of the lower abdomen and the following vital signs: heart rate 102, blood pressure 118/74 mmHg, and respirations 20. Which of the following statements made by the patient would most indicate the presence of pelvic inflammatory disease? A) "My boyfriend was just diagnosed with chlamydia." B) "I just had an IUD inserted." C) "My last menstrual cycle was normal." D) "I have chronic urinary tract infections." ANSWER: A Diff: 2 Page Ref: 11 Standard: Medicine (Gynecology) Objective: 3 9) A 20-year-old sexually active woman presents with severe right-side abdominal pain that radiates to her back. She states that the pain came on sharply during intercourse about 15 minutes earlier, and she reports a small amount of vaginal bleeding. She states that her menstrual cycles have been irregular for the past 3 months. The most likely clinical diagnosis would be: A) ruptured ectopic pregnancy. B) ruptured ovarian cyst. C) spontaneous abortion. D) pelvic inflammatory disease. ANSWER: B Diff: 2 Page Ref: 11-12 Standard: Medicine (Gynecology) Objective: 3 10) A 33-year-old woman presents with a low-grade fever and abdominal pain. She reports that she noticed blood in her urine this morning. Which of the following questions would be most helpful when trying to identify the underlying cause of this patient's symptoms? A) "Have you noticed any foul-smelling discharge?" B) "Do you have any pain or burning with urination?"
  • 10. TO GET ALL CHAPTERS EMAIL ME AT>>>>> donc8246@gmail.com C) "Do you take birth control?"
  • 11. TO GET ALL CHAPTERS EMAIL ME AT>>>>> donc8246@gmail.com D) "Have you vomited today?" ANSWER: B Diff: 3 Page Ref: 12 Standard: Medicine (Gynecology) Objective: 3
  • 12. TO GET ALL CHAPTERS EMAIL ME AT>>>>> donc8246@gmail.com 11) A 19-year-old woman presents with severe lower abdominal pain, an oral temperature of 102.4°F, and skin that is pale and sweaty. She reports that she had an elective abortion 72 hours earlier and has had bloody vaginal discharge ever since. Appropriate treatment for this patient would include which of the following? A) IV fluids, oxygen, and transport B) Position of comfort, pain medication, and delayed transport C) Detailed secondary exam and 12-lead ECG D) Knee-chest position and rapid transport ANSWER: A Diff: 2 Page Ref: 13 Standard: Medicine (Gynecology) Objective: 3 12) A 22-year-old woman presents with severe abdominal pain and signs of shock. When asked whether she is pregnant, she states, "There's no way I'm pregnant, I have an IUD." Which of the following is the most likely cause of her signs and symptoms? A) Endometriosis B) Pelvis inflammatory disease C) Miscarriage D) Ectopic pregnancy ANSWER: D Diff: 2 Page Ref: 12-13 Standard: Medicine (Gynecology) Objective: 3 13) All of the following are associated with spontaneous abortion EXCEPT: A) nontraumatic vaginal bleeding. B) the passage of clots and tissue. C) hypotension and fever. D) cramping abdominal pain. ANSWER: C Diff: 2 Page Ref: 13 Standard: Medicine (Gynecology)
  • 13. TO GET ALL CHAPTERS EMAIL ME AT>>>>> donc8246@gmail.com Objective: 3
  • 14. TO GET ALL CHAPTERS EMAIL ME AT>>>>> donc8246@gmail.com 14) Your patient is a 42-year-old woman who is alert and upset, complaining of vaginal bleeding. She states that the bleeding began ten hours ago and is heavier than her normal menstrual flow. Her last menstrual period was three months ago, G3P3, and there is no other significant gynecologic history. Which of the following is the most likely cause of her signs and symptoms? A) Ectopic pregnancy B) Pelvic inflammatory disease C) Menopause-related dysfunctional uterine bleeding D) Spontaneous abortion ANSWER: D Diff: 2 Page Ref: 13 Standard: Medicine (Gynecology) Objective: 3 15) A 34-year-old woman presents with complaints of vaginal bleeding and pain during intercourse. Her last menstrual period was two weeks ago and was reportedly heavier than normal. She reports that she had saturated two menstrual pads in the past two hours. You should: A) assess vital signs and transport. B) apply oxygen and start a large-bore IV. C) advise her to insert a tampon to control the bleeding. D) ask her if she has a family history of uterine cancer. ANSWER: A Diff: 2 Page Ref: 13 Standard: Medicine (Gynecology) Objective: 4 16) A 16-year-old female patient presents with complaints of abdominal pain and dizziness. She is pale and clammy, and reluctant to answer your questions regarding her sexual activity. Vital signs are: blood pressure 92/62 mmHg, heart rate 102, and respirations 20. You should: A) make it clear that she must be honest about her sexual history. B) start a large-bore IV and transport emergently. C) ask the mother if the patient might be pregnant. D) transport in a position of comfort. ANSWER: B
  • 15. TO GET ALL CHAPTERS EMAIL ME AT>>>>> donc8246@gmail.com Diff: 2 Page Ref: 13 Standard: Medicine (Gynecology) Objective: 4
  • 16. TO GET ALL CHAPTERS EMAIL ME AT>>>>> donc8246@gmail.com 17) Your patient is a 35-year-old woman who is complaining of severe abdominal pain in both lower quadrants. She states that she had a tubal ligation two years ago. Which of the following questions is appropriate to ask during your care of this patient? A) "Have you ever had a sexually transmitted disease?" B) "Do you have more than one sexual partner?" C) "Have you ever had pelvic inflammatory disease (PID)?" D) "When was your last menstrual period?" ANSWER: D Diff: 1 Page Ref: 13 Standard: Medicine (Gynecology) Objective: 4 18) Risk factors for ectopic pregnancy include all of the following EXCEPT: A) use of an intrauterine device for birth control. B) pelvic inflammatory disease. C) previous ectopic pregnancies. D) IV drug usage. ANSWER: D Diff: 2 Page Ref: 12-13 Standard: Medicine (Gynecology) Objective: 3 19) You are called to the scene of a sexual assault. The patient is a 17-year-old female who is crying inconsolably and withdraws when you attempt to touch her. Which of the following is the most appropriate response? A) Explain to the patient that she must allow you to examine her for injuries. B) Ask the patient to describe how she was assaulted so you know where she is injured. C) Tell the patient that you cannot help her if she won't allow you to touch her. D) Explain to the patient that you will not touch her if she does not want you too. ANSWER: D Diff: 1 Page Ref: 14 Standard: Medicine (Gynecology) Objective: 5
  • 17. TO GET ALL CHAPTERS EMAIL ME AT>>>>> donc8246@gmail.com 20) A 35-year-old woman has been raped. She states repeatedly that she wants to change her clothes before going to the hospital, and becomes hysterical when you advise her that she should remain dressed to preserve evidence. You should: A) allow her to change and carefully bag each item of clothing. B) refuse to let her change her clothing, as it will destroy evidence. C) allow her to change only her shirt, not her pants or undergarments. D) promise to let her change as soon as you get to the hospital. ANSWER: A Diff: 2 Page Ref: 14 Standard: Medicine (Gynecology) Objective: 5
  • 18. TO GET ALL CHAPTERS EMAIL ME AT>>>>> donc8246@gmail.com 21) Which of the following is NOT an appropriate adaptation to make in the assessment and care of a sexual assault victim? A) Allow a friend to accompany the victim in the back of the ambulance. B) Ask simple, closed-ended questions about the nature of the assault. C) Keep the back of the ambulance well lit and warm. D) Do not touch the patient unless it is necessary to obtain vital signs or examine injuries. ANSWER: B Diff: 2 Page Ref: 14 Standard: Medicine (Gynecology) Objective: 5 22) A 35-year-old woman presents with abdominal pain that she describes as "cramping and dull." She reports having multiple sexual partners in the past six months, and states that she has an IUD. Vital signs are: blood pressure 118/76 mmHg, heart rate 88, and respirations 18. Appropriate care of this patient includes: A) cardiac monitoring and transport. B) examining the genitalia for hemorrhage. C) elevating the legs during transport. D) position of comfort and routine ALS care. ANSWER: D Diff: 2 Page Ref: 13 Standard: Medicine (Gynecology) Objective: 6 23) A 22-year-old woman presents with left-sided lower abdominal pain. She reports that her last menstrual cycle ended approximately 10 days ago. Her blood pressure is normal and she rates the pain as 6 on a scale of 10. You should: A) palpate the abdomen. B) administer oxygen. C) withhold pain medication. D) look for vaginal bleeding. ANSWER: A Diff: 2 Page Ref: 13
  • 19. TO GET ALL CHAPTERS EMAIL ME AT>>>>> donc8246@gmail.com Standard: Medicine (Gynecology)
  • 20. TO GET ALL CHAPTERS EMAIL ME AT>>>>> donc8246@gmail.com Objective: 6
  • 21. TO GET ALL CHAPTERS EMAIL ME AT>>>>> donc8246@gmail.com 24) Your patient is an 18-year-old woman who is alert and in moderate distress, complaining of abdominal pain and light-headedness. She describes a four-week history of worsening unilateral lower quadrant abdominal pain with onset of malaise, nausea, and vomiting this week. Today, she is experiencing faintness and near-syncope with exertion. She denies pain or difficulty with urination. Her last menstrual period was two weeks ago. She is sexually active and uses oral contraceptives. Physical examination reveals marked tenderness and guarding with palpation of her abdomen. Her skin is cool and diaphoretic. Her vital signs are: heart rate 121, blood pressure 90/58 mmHg, respirations 18. The patient's presentation is most consistent with: A) ectopic pregnancy. B) pelvic inflammatory disease. C) spontaneous abortion. D) pyelonephritis. ANSWER: A Diff: 3 Page Ref: 12-13 Standard: Medicine (Gynecology) Objective: 6 25) A 30-year-old woman presents with a fever of 103.5°F, pale skin, and an altered mental status. She cries out and withdraws when you palpate her abdomen. Her husband reports that she had a cervical biopsy three days earlier, and has be "feeling sick" ever since. Vital signs are: blood pressure 88/60 mmHg, heart rate 110, and respirations 22. You suspect: A) sepsis secondary to endometritis. B) hypovolemia secondary to miscarriage. C) infection of fibroid tumors. D) shock due to pelvic inflammatory disease. ANSWER: A Diff: 3 Page Ref: 12 Standard: Medicine (Gynecology) Objective: 6 26) A 26-year-old woman is complaining of severe abdominal pain and heavy vaginal bleeding. She has used five sanitary napkins in the past hour. Her last menstrual period was six weeks ago, and she describes a history of irregular periods, but never to this extent. Management of this patient
  • 22. TO GET ALL CHAPTERS EMAIL ME AT>>>>> donc8246@gmail.com should include:
  • 23. TO GET ALL CHAPTERS EMAIL ME AT>>>>> donc8246@gmail.com A) IV dextrose. B) oral antiemetic. C) IM epinephrine. D) IV fluids. ANSWER: D Diff: 2 Page Ref: 13 Standard: Medicine (Gynecology) Objective: 6
  • 24. TO GET ALL CHAPTERS EMAIL ME AT>>>>> donc8246@gmail.com 27) Your patient is a 44-year-old woman who is alert and in mild distress. She states that she had an acute onset of sharp, right lower quadrant abdominal pain last evening while having intercourse and that the pain has not subsided. Physical examination reveals tenderness with palpation to the lower right abdominal quadrant. Vital signs are: heart rate 98, respirations 16, blood pressure 116/78 mmHg. She reports she had a tubal ligation ten years ago and that she has several small fibroid tumors. Which of the following is the most likely diagnosis for this patient? A) Ectopic pregnancy B) Pelvic inflammatory disease C) Mittelschmerz D) Ruptured ovarian cyst ANSWER: D Diff: 2 Page Ref: 11-12 Standard: Medicine (Gynecology) Objective: 3 28) Your patient is a transgender man complaining of severe lower abdominal pain. He states that he is in the process of transitioning from female to male, and that he takes hormone supplements. When evaluating this patient, which of the following questions is most appropriate? A) "Are you planning to have gender reassignment surgery?" B) "When was your last menstrual cycle?" C) "How often do you use illegal drugs?" D) "Are you usually sexually active with men, or with women?" ANSWER: B Diff: 3 Page Ref: 13 Standard: Medicine (Gynecology) Objective: 6 29) A 27-year-old woman reports significant vaginal bleeding. She states that she is 11 weeks pregnant and awoke this morning to heavy bleeding including the passage of clots. She is crying and very upset, stating, "I can't lose this baby—I can't live with that." When caring for this patient, you should: A) ask her if she has ever had a miscarriage. B) perform a vaginal exam to determine if she is still bleeding.
  • 25. TO GET ALL CHAPTERS EMAIL ME AT>>>>> donc8246@gmail.com C) transport the patient in a position of comfort and provide emotional support.
  • 26. TO GET ALL CHAPTERS EMAIL ME AT>>>>> donc8246@gmail.com D) inform the patient's husband that she may be having a miscarriage. ANSWER: C Diff: 2 Page Ref: 13 Standard: Medicine (Gynecology) Objective: 6
  • 27. TO GET ALL CHAPTERS EMAIL ME AT>>>>> donc8246@gmail.com 30) A 30-year-old woman has right-sided abdominal pain and is hypotensive. You suspect she may have a ruptured ectopic pregnancy. You recognize: A) life-threatening hemorrhagic shock and the need for fluid resuscitation. B) potentially dangerous obstructive shock that requires rapid transport. C) a common medical condition that affects many women each year. D) a very rare disease process that requires immediate surgery to save the pregnancy. ANSWER: A Diff: 3 Page Ref: 13 Standard: Medicine (Gynecology) Objective: 6 31) Which of the following is NOT an appropriate way to control vaginal bleeding in the prehospital setting? A) Placing sterile gauze over the opening of the vagina B) Using a tampon C) Using a sanitary napkin D) Placing an absorbent pad under the patient ANSWER: B Diff: 1 Page Ref: 13 Standard: Medicine (Gynecology) Objective: 6 32) Your patient is a 36-year-old woman who is alert and complaining of abdominal pain. She states that she is having her period and that this pain is "much different than the cramps I usually get." She describes the pain as achy throughout her pelvis and lower abdomen. She says that this has occurred the past three menstrual cycles and that she has experienced dyspareunia and spotting over the same period. She is G2P2 and has no other significant gynecologic history. Physical examination reveals pain with palpation over her entire abdomen; her skin is warm and dry. Vital signs are: heart rate 84, blood pressure 124/76 mmHg, respiration 12, SpO2 = 99%. Which of the following is the most likely diagnosis for this patient? A) Uterine fibroids B) Endometriosis C) Primary dysmenorrhea
  • 28. TO GET ALL CHAPTERS EMAIL ME AT>>>>> donc8246@gmail.com D) Polycystic ovary disease
  • 29. TO GET ALL CHAPTERS EMAIL ME AT>>>>> donc8246@gmail.com ANSWER: B Diff: 2 Page Ref: 12 Standard: Medicine (Gynecology) Objective: 3
  • 30. TO GET ALL CHAPTERS EMAIL ME AT>>>>> donc8246@gmail.com Chapter 2 Obstetrics 1) Just before an infant is delivered, the fetal scalp can be seen at the vaginal opening during each contraction. This is referred to as: A) effacement. B) crowning. C) dilation. D) presentation. ANSWER: B Diff: 1 Page Ref: 28 Standard: Special Patient Populations (Obstetrics) Objective: 1 2) The blood-rich structure that serves as a lifeline for the developing fetus is called the: A) uterus. B) cervix. C) placenta. D) amniotic sac. ANSWER: C Diff: 1 Page Ref: 20 Standard: Special Patient Populations (Obstetrics) Objective: 1 3) The medically appropriate term for the projected birth date of a baby is: A) probable date of delivery. B) expected date of labor. C) predicted delivery date. D) estimated date of confinement. ANSWER: D Diff: 1 Page Ref: 23 Standard: Special Patient Populations (Obstetrics) Objective: 1
  • 31. TO GET ALL CHAPTERS EMAIL ME AT>>>>> donc8246@gmail.com 4) The term "puerperium" refers to:
  • 32. TO GET ALL CHAPTERS EMAIL ME AT>>>>> donc8246@gmail.com A) the delivery of the afterbirth. B) a specific fertility treatment. C) the time surrounding delivery. D) a technique used to delay contractions. ANSWER: C Diff: 1 Page Ref: 36 Standard: Special Patient Populations (Obstetrics) Objective: 1
  • 33. TO GET ALL CHAPTERS EMAIL ME AT>>>>> donc8246@gmail.com 5) You are caring for a woman who is 32 weeks pregnant. While assessing her breathing, you notice that her rib cage expands visibly with each breath and that her tidal volume appears to be deep. You recognize: A) signs of a severe respiratory condition requiring immediate intervention. B) an increase in tidal volume that is normal during pregnancy. C) signs of obstructive shock. D) an increase in oxygen demand due to an obstetrical emergency. ANSWER: B Diff: 2 Page Ref: 21 Standard: Special Patient Populations (Obstetrics) Objective: 2 6) A 27-year-old pregnant woman was the restrained driver of a vehicle that hit the guardrail at a high rate of speed. She sustained a large laceration to the upper thigh and has lost a significant amount of blood. Vital signs are: heart rate 102, blood pressure 118/78 mmHg, respirations 18, and her skin is pale and clammy. Which of the following BEST explains this patient's presentation? A) Maternal blood volume increases during pregnancy, allowing vital signs to remain normal despite significant blood loss. B) Fetal oxygen demand increases after trauma, causing maternal blood pressure and heart rate to increase immediately after injury. C) Maternal blood volume decreases during pregnancy, amplifying the effects of epinephrine and allowing for the maintenance of blood pressure. D) Maternal vital signs remain normal following trauma to ensure the survival of the developing fetus. ANSWER: A Diff: 2 Page Ref: 22 Standard: Special Patient Populations (Obstetrics) Objective: 2 7) A 30-year-old woman complains of swelling in both of her legs. She is 32 weeks pregnant, and states that she has also developed varicose veins over the past few weeks. Which of the following BEST explains the cause of this patient's signs and symptoms? A) The gravid uterus compresses the superior vena cava, decreasing venous return to the heart.
  • 34. TO GET ALL CHAPTERS EMAIL ME AT>>>>> donc8246@gmail.com B) Changes in the endocrine system cause peripheral edema during the late stages of pregnancy. C) The gravid uterus compresses the pelvic and femoral vessels, decreasing venous return and causing venous stasis. D) Changes in kidney function alter the normal electrolyte balance, causing tissue edema in the legs and feet. ANSWER: C Diff: 2 Page Ref: 22 Standard: Special Patient Populations (Obstetrics) Objective: 2
  • 35. TO GET ALL CHAPTERS EMAIL ME AT>>>>> donc8246@gmail.com 8) During which period of development is the fetus most susceptible to damage from maternal exposure to toxins, such as alcohol and tobacco? A) 1-8 weeks B) 16-20 weeks C) 8-12 weeks D) 20-40 weeks ANSWER: C Diff: 2 Page Ref: 24 Standard: Special Patient Populations (Obstetrics) Objective: 2 9) Which structure allows oxygenated maternal blood to bypass the uninflated lungs of the developing fetus? A) Ductus venosus B) Foramen ovale C) Umbilical artery D) Ductus arteriosus ANSWER: A Diff: 2 Page Ref: 25 Standard: Special Patient Populations (Obstetrics) Objective: 2 10) A 38-year-old pregnant woman called 911 after she developed a "splitting headache" unrelieved by over-the-counter analgesics. She states that she has a previous diagnosis of diabetes that is typically well controlled with insulin injections. This patient's history of diabetes makes her more likely to develop which of the following pregnancy-related conditions? A) Preeclampsia B) Cerebral aneurysm C) Congestive heart failure D) Stroke ANSWER: A
  • 36. TO GET ALL CHAPTERS EMAIL ME AT>>>>> donc8246@gmail.com Diff: 2 Page Ref: 26 Standard: Special Patient Populations (Obstetrics)
  • 37. TO GET ALL CHAPTERS EMAIL ME AT>>>>> donc8246@gmail.com Objective: 3
  • 38. TO GET ALL CHAPTERS EMAIL ME AT>>>>> donc8246@gmail.com 11) A 29-year-old woman is 28 weeks pregnant. She complains of nausea, abdominal pain, and right shoulder pain. She states she vomited once, which caused temporary relief; however, the pain returned shortly afterward. She has no pertinent medical history or allergies, has stable vital signs, and states that she ate a cheeseburger about 20 minutes prior to the start of her pain. The most likely cause of this patient's presentation is: A) food poisoning. B) cholecystitis. C) hyperemesis gravidarum. D) appendicitis. ANSWER: B Diff: 2 Page Ref: 29 Standard: Special Patient Populations (Obstetrics) Objective: 3 12) You are caring for a 42-year-old woman who is 32 weeks pregnant with twins. She is complaining of a severe headache and blurred vision, and lives in a rural area approximately 30 miles from the nearest ED. Your physical exam reveals significant peripheral edema and the following vital signs: heart rate 98, blood pressure 156/98 mmHg, respirations 18. The patient states she has a history of hypertension, but has not taken her prescribed medications because "they aren't safe for the babies." Appropriate treatment for this patient includes: A) administration of nitroglycerine and delayed transport. B) intravenous calcium chloride and 12-lead ECG. C) administration of aspirin and a prehospital stroke assessment. D) intravenous magnesium sulfate and rapid transport. ANSWER: D Diff: 3 Page Ref: 32-33 Standard: Special Patient Populations (Obstetrics) Objective: 3 13) A 24-year-old pregnant woman is actively seizing. Her partner reports that she complained of abdominal pain approximately 10 minutes ago, and then began seizing. He reports that the seizure activity has been consistent for the past 10 minutes, and that she has no significant medical history. You should:
  • 39. TO GET ALL CHAPTERS EMAIL ME AT>>>>> donc8246@gmail.com A) apply a nasal cannula, assess blood glucose level, and transport in the left lateral position.
  • 40. TO GET ALL CHAPTERS EMAIL ME AT>>>>> donc8246@gmail.com B) insert a nasal airway, administer midazolam, and obtain vital signs. C) apply high-flow oxygen, administer magnesium sulfate, and transport emergently. D) suction the airway, obtain IV access, and assess blood pressure. ANSWER: C Diff: 2 Page Ref: 32-33 Standard: Special Patient Populations (Obstetrics) Objective: 3
  • 41. TO GET ALL CHAPTERS EMAIL ME AT>>>>> donc8246@gmail.com 14) A 25-year-old woman presents complaining of heavy vaginal bleeding accompanied by cramping abdominal pain and backache. She can't stop crying and reports that she is 11 weeks pregnant with her first child. Your physical exam reveals what appears to be the fetus and umbilical cord passing through the vaginal opening; however, the placenta has not been passed. Appropriate care for this patient includes: A) clamping and cutting the umbilical cord, wrapping fetal material in linen, and providing emotional support for the patient. B) leaving the fetus untouched and transporting the patient in a position of comfort. C) oxygen, 1000 mL fluid bolus, and rapid transport. D) disposing of the fetal material in a biohazard bag, left lateral position, and providing emotional support for the patient. ANSWER: A Diff: 3 Page Ref: 29 Standard: Special Patient Populations (Obstetrics) Objective: 4 15) You are caring for a patient who is 36 weeks pregnant. She states that she has been experiencing vaginal bleeding after intercourse, and denies any associated pain. Vital signs are: heart rate 84, respirations 18, blood pressure 102/72 mmHg, and warm, dry skin. The most likely cause of this patient's vaginal bleeding is: A) spontaneous abortion. B) placenta previa. C) abruptio placentae. D) false labor. ANSWER: B Diff: 2 Page Ref: 31 Standard: Special Patient Populations (Obstetrics) Objective: 3 16) A 37-year-old, obviously pregnant woman complains of sharp, tearing abdominal pain. She reports that she is pregnant with her fifth child, but states she cannot remember the exact due date. The patient has a history of drug use and states that she last smoked crack cocaine approximately 30 minutes ago. You suspect:
  • 42. TO GET ALL CHAPTERS EMAIL ME AT>>>>> donc8246@gmail.com A) spontaneous abortion.
  • 43. TO GET ALL CHAPTERS EMAIL ME AT>>>>> donc8246@gmail.com B) placenta previa. C) abruptio placentae. D) false labor. ANSWER: C Diff: 2 Page Ref: 31-32 Standard: Special Patient Populations (Obstetrics) Objective: 3
  • 44. TO GET ALL CHAPTERS EMAIL ME AT>>>>> donc8246@gmail.com 17) A pregnant woman is complaining of a severe headache and "feeling ill." Her vital signs are: heart rate 92, blood pressure 120/82 mmHg, and respiratory rate of 16. Which of the following questions would be most helpful in identifying a possible pregnancy-related emergency? A) "Do you have a history of migraine headaches?" B) "Have you been experiencing morning sickness or excessive vomiting today?" C) "When was the last time you saw your doctor?" D) "What has been a typical blood pressure for you during this pregnancy?" ANSWER: D Diff: 2 Page Ref: 33 Standard: Special Patient Populations (Obstetrics) Objective: 4 18) Which of the following techniques is an appropriate method of estimating the gestational age? A) Measuring the circumference of the woman's abdomen; each inch corresponds to one week of pregnancy. B) Palpate the abdomen and estimate gestational age based on the size of the developing fetus. C) Measure the height of the fundus; each centimeter corresponds to one week of pregnancy. D) Palpate the abdomen; if fetal movement is felt then the pregnancy is at least 14 weeks. ANSWER: C Diff: 2 Page Ref: 47 Standard: Special Patient Populations (Obstetrics) Objective: 4 19) When assessing the vital signs of a pregnant patient, it is generally most appropriate to have the patient in which position? A) Supine B) Left lateral recumbent C) Semi-Fowler's D) Standing ANSWER: B
  • 45. TO GET ALL CHAPTERS EMAIL ME AT>>>>> donc8246@gmail.com Diff: 1 Page Ref: 27 Standard: Special Patient Populations (Obstetrics)
  • 46. TO GET ALL CHAPTERS EMAIL ME AT>>>>> donc8246@gmail.com Objective: 4 20) You are caring for a 19-year-old woman who states that she is 32 weeks pregnant. She is experiencing cramping abdominal pain every six to seven minutes and reports a brief period of fluid discharge from her vagina approximately 15 minutes ago. You should: A) provide routine BLS care and transport. B) begin a fluid bolus and transport code 3. C) administer ondansetron and reassess vital signs. D) apply oxygen and perform a physical exam. ANSWER: B Diff: 2 Page Ref: 35-36 Standard: Special Patient Populations (Obstetrics) Objective: 5
  • 47. TO GET ALL CHAPTERS EMAIL ME AT>>>>> donc8246@gmail.com 21) The second stage of labor begins: A) with the complete dilation of the cervix. B) at the onset of contractions. C) when the baby is delivered. D) at the rupture of membranes. ANSWER: A Diff: 2 Page Ref: 36 Standard: Special Patient Populations (Obstetrics) Objective: 6 22) You are caring for a woman in labor. She has had three previous deliveries and states that her contractions are approximately 3 minutes apart. Physical exam reveals a bulging of tissue at the vaginal opening with each contraction; however, the baby's head is not yet visible. You should: A) encourage the mother to breathe and ready her for transport. B) prepare for imminent delivery. C) place the mother in the knee-chest position. D) perform an internal exam to see how much the cervix is dilated. ANSWER: B Diff: 2 Page Ref: 38 Standard: Special Patient Populations (Obstetrics) Objective: 6 23) What is the appropriate sequence of care for a newborn following a normal delivery? A) Suction the nose, dry the infant, and cut the cord. B) Dry the infant, cut the cord, and place infant on mother's stomach. C) Suction the mouth, deliver the placenta, and cut the cord. D) Apply oxygen, dry the infant, and give the infant to the mother. ANSWER: B Diff: 2 Page Ref: 39, 40 Standard: Special Patient Populations (Obstetrics) Objective: 7
  • 48. TO GET ALL CHAPTERS EMAIL ME AT>>>>> donc8246@gmail.com 24) A newborn has just been delivered. He is centrally pink with pale extremities, has a heart rate of 110, and is actively crying and moving. The appropriate APGAR score for this infant is:
  • 49. TO GET ALL CHAPTERS EMAIL ME AT>>>>> donc8246@gmail.com A) 6. B) 7. C) 8. D) 9. ANSWER: D Diff: 2 Page Ref: 41 Standard: Special Patient Populations (Obstetrics) Objective: 7
  • 50. TO GET ALL CHAPTERS EMAIL ME AT>>>>> donc8246@gmail.com 25) A newborn is listless and has a heart rate of 50 beats per minute after 30 seconds of stimulation. You should FIRST: A) initiate positive pressure ventilation. B) begin chest compressions. C) obtain IV access. D) apply the AED. ANSWER: A Diff: 2 Page Ref: 41 Standard: Special Patient Populations (Obstetrics) Objective: 7 26) A 42-year-old woman is 38 weeks pregnant and in active labor. While assessing for crowning, you notice that although the top of the baby's head is visible, a loop of the umbilical cord is protruding from the vagina. You should: A) gently attempt to reinsert the cord into the vaginal canal. B) encourage the mother to push while gently pulling traction. C) clamp and cut the presenting section of the umbilical cord. D) insert two fingers to raise the head of the baby off the cord. ANSWER: D Diff: 2 Page Ref: 43 Standard: Special Patient Populations (Obstetrics) Objective: 8 27) You are assisting a mother with an out-of-hospital delivery. After the head delivers, it immediately retracts back into the perineum. You should: A) transport in the knee-chest position. B) instruct the mother to drop her buttocks off the end of the bed. C) gently pull downward on the infant's head. D) instruct the mother to avoid pushing if possible. ANSWER: B Diff: 2 Page Ref: 43 Standard: Special Patient Populations (Obstetrics)
  • 51. TO GET ALL CHAPTERS EMAIL ME AT>>>>> donc8246@gmail.com Objective: 8
  • 52. TO GET ALL CHAPTERS EMAIL ME AT>>>>> donc8246@gmail.com 28) During delivery, you notice a yellowish-green fluid on the baby's head and face. You recognize: A) a normal side effect of delivery. B) that emergency transport is required. C) a sign of fetal hypoxia. D) that the infant will require resuscitation. ANSWER: C Diff: 2 Page Ref: 45 Standard: Special Patient Populations (Obstetrics) Objective: 8
  • 53. TO GET ALL CHAPTERS EMAIL ME AT>>>>> donc8246@gmail.com 29) You have just delivered a healthy newborn to a 35-year-old woman. There is a steady flow of blood from the vagina and her uterus feels soft upon palpation. Her vital signs are: heart rate 122, respirations 20, blood pressure 90/60 mmHg. Appropriate management of this patient includes: A) sanitary pads placed over the vagina and continued monitoring. B) two large-bore IVs and oxytocin administration. C) Trendelenberg position and administration of magnesium sulfate. D) low-flow oxygen and position of comfort. ANSWER: B Diff: 2 Page Ref: 45 Standard: Special Patient Populations (Obstetrics) Objective: 8 30) A 32-year-old woman was the restrained passenger of a vehicle that hit a tree at a high rate of speed. She is 34 weeks pregnant and complains of excruciating abdominal pain. She is cool and diaphoretic and has a faint and rapid radial pulse. You suspect: A) traumatic onset of labor. B) uterine inversion. C) uterine rupture. D) premature rupture of membranes. ANSWER: C Diff: 2 Page Ref: 46 Standard: Special Patient Populations (Obstetrics) Objective: 8 31) A woman late in her pregnancy is in cardiac arrest. When resuscitating this patient, it is appropriate to: A) compress at a rate of at least 120 per minute. B) use a mechanical compression device. C) lift and push the gravid uterus to the left. D) assess for imminent delivery. ANSWER: C Diff: 2 Page Ref: 47-48
  • 54. TO GET ALL CHAPTERS EMAIL ME AT>>>>> donc8246@gmail.com Standard: Special Patient Populations (Obstetrics) Objective: 8
  • 55. TO GET ALL CHAPTERS EMAIL ME AT>>>>> donc8246@gmail.com 32) You have just delivered an infant at 33 weeks gestation. The infant is blue and limp and has a brachial pulse of 70 beats per minute. What is the appropriate sequence of care for this infant? A) Keep warm and dry, chest compressions, clamp and cut the cord. B) Keep warm and dry, clamp and cut the cord, positive pressure ventilations. C) Clamp and cut the cord, positive pressure ventilations, keep warm and dry. D) Clamp and cut the cord, chest compressions, positive pressure ventilations. ANSWER: B Diff: 2 Page Ref: 41 Standard: Special Patient Populations (Obstetrics) Objective: 8
  • 56. TO GET ALL CHAPTERS EMAIL ME AT>>>>> donc8246@gmail.com 33) A 21-year-old woman is complaining of abdominal pain. She says she is 25 weeks pregnant and has been actively trying to stop using methamphetamines, but slipped and used earlier today. Her abdominal pain comes every 8 to 10 minutes and is described as sharp and cramping. You suspect: A) hypertensive disorder of pregnancy. B) abruptio placentae. C) toxic shock syndrome. D) preterm labor. ANSWER: D Diff: 2 Page Ref: 35 Standard: Special Patient Populations (Obstetrics) Objective: 8 34) When is the ideal time to assess a neonate for the APGAR score? A) 2 to 3 minutes after delivery B) 1 minute and 5 minutes after delivery C) 5 to 10 minutes after delivery D) 2 minutes and 10 minutes after delivery ANSWER: B Diff: 2 Page Ref: 40 Standard: Special Patient Populations (Obstetrics) Objective: 7 Chapter 3 Neonatology 1) In some cases, the spinal cord and associated structures of a newborn may be exposed. This abnormality is called a: A) Tetralogy of Fallot. B) meningomyelocele. C) omphalocele. D) choanal atresia. ANSWER: B Diff: 2 Page Ref: 59
  • 57. TO GET ALL CHAPTERS EMAIL ME AT>>>>> donc8246@gmail.com Standard: Special Patient Populations (Neonatal Care)
  • 58. TO GET ALL CHAPTERS EMAIL ME AT>>>>> donc8246@gmail.com Objective: 1 2) Which of the following best describes Pierre Robin syndrome? A) A congenital condition characterized by a small jaw and large tongue in conjunction with a cleft palate B) A condition in which the ductus arteriosis fails to close completely C) Repeat febrile seizures within the first 6 months of life D) Persistent pneumonia following meconium aspiration ANSWER: A Diff: 2 Page Ref: 59 Standard: Special Patient Populations (Neonatal Care) Objective: 1
  • 59. TO GET ALL CHAPTERS EMAIL ME AT>>>>> donc8246@gmail.com 3) Which of the following is TRUE regarding vaginal delivery? A) Infants delivered vaginally are at greater risk of post-delivery complications. B) The compression of the infant's chest during vaginal delivery aids in the removal of fluid from the fetal lungs. C) Vaginal delivery stimulates the production of insulin in the newborn. D) The foramen ovale close more successfully in newborns delivered vaginally. ANSWER: B Diff: 2 Page Ref: 55 Standard: Special Patient Populations (Neonatal Care) Objective: 2 4) Factors that stimulate the newborn to take the first breath include all of the following EXCEPT: A) hypothermia. B) hypoxia. C) hyperglycemia. D) acidosis. ANSWER: C Diff: 2 Page Ref: 55 Standard: Special Patient Populations (Neonatal Care) Objective: 3 5) You are caring for a newborn who had prolonged respiratory distress and hypoxia following birth. You recognize that continued hypoxia and subsequent severe acidosis can cause: A) refractory hypotension. B) rapid development of fetal alveoli to compensate for hypoxia. C) persistent fetal circulation. D) the formation of congenital heart defects. ANSWER: C Diff: 3 Page Ref: 55 Standard: Special Patient Populations (Neonatal Care)
  • 60. TO GET ALL CHAPTERS EMAIL ME AT>>>>> donc8246@gmail.com Objective: 2
  • 61. TO GET ALL CHAPTERS EMAIL ME AT>>>>> donc8246@gmail.com 6) You have been called to the residence of a five-day-old infant. His parents are concernedbecause he sleeps about 18 hours a day. They took a rectal temperature and got a reading of100°F. Which of the following is the best course of action? A) Administer a 10 mL/kg fluid bolus. B) Undress the infant to his diaper and sponge him with tepid water. C) Reassure the parents that the infant's sleep patterns and temperature are normal. D) Contact medical control to order acetaminophen. ANSWER: C Diff: 2 Page Ref: 59-60 Standard: Special Patient Populations (Neonatal Care) Objective: 3
  • 62. TO GET ALL CHAPTERS EMAIL ME AT>>>>> donc8246@gmail.com 7) You have just assisted in the delivery of a full-term infant in the back of the ambulance. Ten minutes after delivery, you obtain vital signs on the newborn and find the following: heart rate 120, respirations 54, and SpO2 of 90%. You recognize: A) the need for immediate supplemental oxygen. B) appropriate vital signs for this infant. C) tachycardia and the possibility of shock. D) tachypnea and possible respiratory distress. ANSWER: C Diff: 2 Page Ref: 60 Standard: Special Patient Populations (Neonatal Care) Objective: 5 8) Which of the following is NOT an option for prehospital vascular access in the newborn? A) Cannulation of a peripheral vein in an extremity B) Intraosseous needle placed in the proximal tibia C) Femoral vein cannulation D) Umbilical vein cannulation ANSWER: C Diff: 2 Page Ref: 67 Standard: Special Patient Populations (Neonatal Care) Objective: 7 9) Which of the following is NOT a recommended method for assessing the heart rate of a newborn? A) Palpating the carotid pulse B) Palpate the umbilical pulse C) Auscultate heart sounds D) Palpate a femoral pulse ANSWER: A Diff: 2 Page Ref: 64-65 Standard: Special Patient Populations (Neonatal Care) Objective: 5
  • 63. TO GET ALL CHAPTERS EMAIL ME AT>>>>> donc8246@gmail.com 10) Which of the following statements is TRUE regarding the cardiac output of a neonate?
  • 64. TO GET ALL CHAPTERS EMAIL ME AT>>>>> donc8246@gmail.com A) Cardiac output doubles within the first few hours of life. B) The cardiac output of a neonate is strongly influenced by body temperature. C) Neonates do not readily alter their pulse rate; cardiac output depends mainly on stroke volume. D) Newborns have a fixed stroke volume; cardiac output depends mostly on heart rate. ANSWER: D Diff: 2 Page Ref: 64 Standard: Special Patient Populations (Neonatal Care) Objective: 3
  • 65. TO GET ALL CHAPTERS EMAIL ME AT>>>>> donc8246@gmail.com 11) A newborn has just been delivered. You notice a greenish-brown substance on the infant, and observe a vigorous cry and active movement. What is the appropriate way to manage this patient? A) Prepare to suction the trachea with an endotracheal tube and meconium aspirator. B) Using a bulb syringe, suction the nose and then the mouth. C) Rub the infant with a dry towel and place on the mother's chest. D) Lay the infant supine and place padding behind the shoulders. ANSWER: C Diff: 2 Page Ref: 70-71 Standard: Special Patient Populations (Neonatal Care) Objective: 3 12) At 90 seconds after birth, based on the following findings, which newborn does NOT require resuscitative efforts beyond routine care? A) HR = 100, RR = 30, peripheral cyanosis, APGAR = 8 B) HR = 158, RR = 40, central cyanosis, APGAR = 6 C) HR = 75, RR = 25, central cyanosis, APGAR = 5 D) HR = 98, RR = 27, peripheral cyanosis, APGAR = 6 ANSWER: A Diff: 2 Page Ref: 64-65 Standard: Special Patient Populations (Neonatal Care) Objective: 3 13) You have just assisted in the delivery of a 34-week-gestation infant in the office of a methadone clinic. After 30 seconds of drying and stimulation, the infant remains limp and cyanotic, with a pulse of 50. Which of the following is the most appropriate action to take at this point? A) Begin CPR at a rate of 3 compressions to 1 ventilation. B) Intubate the trachea, ventilate, reassess, and prepare to transport to a NICU. C) Begin bag-valve-mask ventilations and establish intravenous access. D) Continue to warm and dry the infant for an additional 30 seconds, while preparing to intubate. ANSWER: A Diff: 3 Page Ref: 60 Standard: Special Patient Populations (Neonatal Care)
  • 66. TO GET ALL CHAPTERS EMAIL ME AT>>>>> donc8246@gmail.com Objective: 5 14) Immediately after delivery, a neonate is crying and moving his arms and legs, but is centrally cyanotic. You recognize: A) the need for supplemental oxygen. B) signs of a normal, healthy infant. C) the need to prevent further heat loss. D) signs of a congenital heart abnormality. ANSWER: B Diff: 2 Page Ref: 60-61 Standard: Special Patient Populations (Neonatal Care) Objective: 3
  • 67. TO GET ALL CHAPTERS EMAIL ME AT>>>>> donc8246@gmail.com 15) All of the following are accurate methods for obtaining the heart rate of a newborn EXCEPT: A) a pulse oximeter. B) 3-lead ECG. C) palpating a brachial pulse. D) auscultating heart tones. ANSWER: C Diff: 2 Page Ref: 64-65 Standard: Special Patient Populations (Neonatal Care) Objective: 5 16) You are resuscitating a 15-day-old infant who was found pulseless and apneic in her crib. Which of the following questions would be most helpful when obtaining a history from the parents? A) "Does your daughter have any congenital heart abnormalities?" B) "Was your baby sleeping face up?" C) "Is there a family history of heart disease?" D) "Have any of your other children died from SIDS?" ANSWER: A Diff: 3 Page Ref: 76 Standard: Special Patient Populations (Neonatal Care) Objective: 5 17) You are called to the home of a 72-hour-old infant whose mother is concerned because the infant has been "vomiting after she eats." Based on this information, which of the following should you include in your history and physical assessment? A) End-tidal CO2 monitoring B) Auscultation of gastric sounds C) Asking if the baby could have ingested any toxins D) Assessing the fontanels ANSWER: D Diff: 3 Page Ref: 75 Standard: Special Patient Populations (Neonatal Care)
  • 68. TO GET ALL CHAPTERS EMAIL ME AT>>>>> donc8246@gmail.com Objective: 5
  • 69. TO GET ALL CHAPTERS EMAIL ME AT>>>>> donc8246@gmail.com 18) You have just assisted in the delivery of an approximately 4 kg newborn whose mother is a rather petite primapara. On assessment, you note that there is no spontaneous movement of the infant's right arm at the shoulder, but he moves the elbow and wrist. He otherwise exhibits vigorous movement and has a 1-minute APGAR score of 9. Which of the following should you suspect? A) Neonatal abstinence syndrome B) Spinal cord damage C) Fractured clavicle D) Caput succedaneum ANSWER: C Diff: 2 Page Ref: 76 Standard: Special Patient Populations (Neonatal Care) Objective: 6
  • 70. TO GET ALL CHAPTERS EMAIL ME AT>>>>> donc8246@gmail.com 19) You are assessing a 1-minute APGAR score on a patient who has peripheral cyanosis and a heart rate of 98, who cried spontaneously after delivery, is actively moving his extremities, and has a strong cry. What is the APGAR score of this patient? A) 9 B) 7 C) 8 D) 10 ANSWER : C Diff: 2 Page Ref: 61 Standard: Special Patient Populations (Neonatal Care) Objective: 6 20) A newborn was found pulseless and apneic in his crib. After lengthy attempts at resuscitation, medical control has advised you to stop resuscitation. When you inform the parents, they become hysterical and the mother cries, "Give me my baby! I don't want you touching him!" You should: A) gently tell the mother that, unfortunately, she cannot hold the baby until the coroner arrives. B) hand the child to the mother and allow the parents time alone with the child. C) request that a law enforcement officer remain with the mother. D) move the infant to the ambulance and transport. ANSWER: B Diff: 2 Page Ref: 73 Standard: Special Patient Populations (Neonatal Care) Objective: 5 21) You are called to a residence to care for a 3-hour-old infant in respiratory distress. Your physical exam reveals retractions, grunting, and tachypnea, along with rhonchi and crackles in the lungs. The midwife reports that the amniotic fluid had "thin meconium." Which of the following is the most likely cause of this infant's signs and symptoms? A) Tetralogy of Fallot B) Cardiac compromise C) Persistent fetal circulation
  • 71. TO GET ALL CHAPTERS EMAIL ME AT>>>>> donc8246@gmail.com D) Meconium aspiration